Anda di halaman 1dari 169

iii

MAT1512/1/2010-2012

Contents
Introduction
How to use the study guide
Keys to success in studying Mathematics
Preparing for the examination
CHAPTER 0:

Preliminaries

Page
vii
viii
ix
ix
1

1. Background

2. Learning outcomes

3. Principles of problem solving

4. Summary

5. The way forward

CHAPTER 1:

Limits and Continuity

1. Background

2. Learning outcomes

3. Prescribed reading

4. The limit

4.1 Introduction to the limit concept

4.2 Definition of a limit: left- and right-hand limits

5. Worked examples
I. Limits as x c (c R)

II. Limits as x

6
7
10

III. Limits involving absolute values

14

IV. Left-hand and right-hand limit

16

V. Limits involving trigonometric functions


VI. The Squeeze Theorem
VII. The definition of a limit

VIII. Continuity
Key points

19
23
26
29
38

iv
CHAPTER 2:

Dierentiation of dierent types of functions

39

1. Background

39

2. Learning outcomes

39

3. Prescribed reading

40

4. The deriviative

40

4.1 Introducing the derivative

40

4.2 Definition of the derivative

42

5. Worked examples
I. Dierentiation from first principles (definition of the derivative)
II. Basic dierentiation formulas

42
44
46

(a) The power rule

46

(b) The product rule

47

(c) The quotient rule

48

(d) The chain rule

49

(e) Combinations of rules

50

III. Derivatives of trigonometric functions and inverse trigonometric functions

52

(a) Derivatives of trigonometric functions

52

(b) Derivatives of inverse trigonometric functions

56

IV. Derivatives of exponential and logarithmic functions

57

(a) The exponential function

57

(b) The logarithmic function

58

(c) Examples of the exponential function

59

(d) Examples of the logarithmic function

60

V. Logarithmic dierentiation

62

(a) The simplification of functions

62

(b) Functions of the form f (x) = g(x)h(x)

64

VI. Implicit dierentiation

66

VII. Tangents and normal lines

71

VIII. The Mean Value Theorem

78

Key points

81

v
CHAPTER 3:

MAT1512/1/2010-2012

Integration

82

1. Background

82

2. Learning outcomes

82

3. Prescribed reading

83

4. Worked examples

83

I. Antiderivatives
II. The definite integral and the Fundamental Theorem of Calculus Part I

84
86

III. The definite integral and the area between the curve and the x-axis

87

IV. The definite integral and the area under the curve

90

V. The Mean Value Theorem for definite integrals

94

VI. The Fundamental Theorem of Calculus Part II

96

VII. Integration in general


VIII. Indefinite integrals

99
100

IX. Integration by substitution

101

(a) Indefinite integrals

101

(b) Definite integrals

106

(c) Steps for integration by substitution

108

X. Integration of exponential and logarithmic functions

109

XI. Review of formulas and techniques of integration <Author: This heading does not appear in
the text. Please insert in the text. CP>
Key points

115

CHAPTER 4:

Firstorder Dierential Equations, Growth and Decay


and Partial derivatives

116

1. Background

116

2. Learning outcomes

116

3. Prescribed reading

117

4. Worked examples

117

I. First-order dierential equations

117

(a) The solution of dierential equations

117

(b) Separable dierential equations

117

(c) Initial-value problems

122

vi
II. Growth and decay

126

III. Partial derivatives

135

(a) The partial derivatives technique

135

(b) Notations for partial derivatives

136

(c) Rules for finding the partial derivatives of z = f (x, y)

136

(d) Partial dierentiation

138

(e) Functions of more than two variables

139

(f) Higher-order derivatives

140

Key points

149

APPENDIX

150

I. Sequences and Summation (Sigma) Notation


II. Mathematical Induction
III. The Binomial Theorem

150
155
159

vii

MAT1512/1

Introduction
Calculus is a set of formal rules and procedures. It gives you the tools you need to measure change both qualitatively and quantitatively. Wikipedia (www.wikepedia.org) defines calculus as a branch of mathematics that includes
the study of limits, derivatives, integrals, and infinite series, which constitutes a major part of modern university education. Calculus has widespread applications in science and engineering and is used to solve complex and expansive
problems for which algebra alone is insufficient. It builds on analytical geometry and mathematical analysis and includes two major branches differential calculus and integral calculus which are related through the Fundamental
Theorem of Calculus. Differential calculus explores and analyses rates of change quantitatively and qualitatively.
Integral calculus deals with the analysis (quantitatively and/or qualitatively) of how quantities or measures of values
accumulate or diminish over time. The two processes differentiation and integration are reciprocal.
The purpose of this module is to equip you, the learner, with those basic skills in differential and integral calculus that are essential for the physical, life, and economic sciences. Most of the time you will be dealing with
functions. Basically, a function is a generalised input-output process that defines the mapping of a set of input
values to a set of output values. It is often defined as a rule for obtaining a numerical value from another given
numerical value. You are also going to have to develop a very large repertoire of methods for depicting functions
graphically/geometrically.
This course is built around your prescribed book. The purpose of this study guide is to guide you through those
parts of the prescribed book that you must study for this module, and to provide you with many additional worked
examples. The prescribed book is:

Robert T Smith & Roland B Minton


Calculus, Early Transcendental Functions
McGraw-Hill Higher Education International Edition
Third edition, c2007
ISSBN 13:978007110715-8

viii

How to use the study guide


From now on we will refer to the prescribed book as Smith and Minton, or simply Smith. The study guide
must always be used in conjunction with the prescribed textbook, because it is not a complete set of notes on the
book. Chapters 1 to 4 of this study guide contain many additional worked problems, taken from past examination
papers and assignments. Before going through them, study the relevant parts in Smith, and do the examples and
some of the exercises in Smith. Also, before going through our solution to a problem in the study guide, try solving
it yourself. Remember that reading maths often means reading the same thing over and over again.
We have included numerous worked examples for you. These are designed to stimulate your thinking in such a
way that you will come to appreciate and master the delicate beauty and intricacies of the subject. All you have to
do is keep on going! Follow all the instructions given. Try to write down all the answers to the activities in full.
This is extremely important, as a major part of learning mathematics is thinking and writing down what you think.
By writing everything down, you will develop the essential skill of communicating mathematics effectively. The
other reason for writing your answers down, is to prevent you from losing your train of thought about a concept
or mathematical idea. If this happens, it takes a while to get your reasoning back to the same point. If you have
everything written down, you can also go back the next day and check your reasoning. Learning mathematics is an
activity, and you will only learn by doing. Because you will be thinking about the problems, you will, in most cases,
be able to determine by yourself whether you are right or not.
Your assignments are included in Tutorial Letter 101. Attempt these once you have completed all the related
activities in your study guide. Spend a part of your time each day doing some of the questions, and a part studying
new material. Being able to do an assignment is proof that you have mastered the work of that particular section. In
Tutorial Letter 101 we have indicated which assignments you should submit for evaluation.
Also remember that statements, theorems and definitions are the building blocks of your mathematical language
you cannot learn anything without knowing the basic facts. Begin by reading the preface of your prescribed book.
This should give you a good idea of the importance of the subject you are about to study.

ix

MAT1512/1

Keys to success in studying Mathematics


Plan

Plan each week according to what happens in your life that week. Decide how many
study hours you need, and decide where you are going to fit those hours in. Write
down what you plan to do in each session. This way you can look forward to the
work you have set for yourself and measure your success.

Evaluate

Evaluate each study session. Did you enjoy your studies? If not, why not? What
can you change to make it better? Did you achieve what you set out to achieve?
Did you use your time constructively? What can you do to improve your time
management?

Understand

Understand the way you learn. One can only learn maths through repetition and
practice. DO IT, even if you think you know the answer. Understanding comes
through repetition, and this understanding eventually brings the JOY of mastering
mathematics.

Structure

Structure each study session for best results. A suggestion is to divide your time into
four parts: (1) Do 5 to 10 minutes revision and read through the important points.
(2) Go through all the questions of the previous activities. (3) Study new material
by following the instructions in the study guide. (4) Attempt the questions in your
assignments about the sections you have completed.

Get organised

(1) Make notes of the facts you have to remember. (2) Write down the answers to
all your activities. (3) Complete all the ASSIGNMENTS!

Pace yourself

Work out how fast you will have to go. Some sections will take longer than others,
but assign about one month for each chapter. This will give you 2 to 4 days for each
section in each chapter. Bear in mind that this is not the only module you have
registered for.

Preparing for the examination


You are studying to improve yourself and NOT only to pass the examination. The examination is there to REWARD
you for what you have done and to CONFIRM that you have done your work properly. If you have followed all
the instructions above, you will already be prepared for the examination and have nothing to worry about. Many
students spend their time thinking that they cannot master mathematics. This is not true. If you put your mind to it
and work hard, you will eventually master this subject.

GOOD LUCK!

MAT1512/1

CHAPTER 0
PRELIMINARIES
1.

Background

This chapter of the study guide deals with Chapter 0 in Smith. It forms a link between school mathematics and calculus. Topics in algebra, trigonometry, analytic geometry and functions which are needed for calculus are reviewed.
Graphs of functions are studied, since they enable us to see and understand the behaviour of functions. In studying
this chapter, you should brush up and revise your school mathematics especially those parts that are needed for
an effective study of calculus. The material in this chapter is background material. At the same time, some topics
which are important to understand calculus are introduced. Therefore, the chapter will contain a few topics which
you may have touched on only briefly, or not at all, at school.
2.

Learning outcomes

At the end of this chapter, you should be able to

algebraically manipulate real numbers, solve equations and work comfortably with mathematical concepts
such as variables, inequalities and absolute values
recognise, demonstrate your knowledge of and work with the different types of functions (polynomial, rational, trigonometric, exponential and logarithmic functions), their properties and their representations

In calculus, the basic mathematical tool at the root of all we do is the function. That is why, in this preliminary
chapter, you have to familiarise yourself with mathematical ideas related to functions, such as the different types of
functions, their graphs and representations of how they behave. First read the preliminaries of this chapter given on
page 1 in Smith. You should study the whole chapter. Remember that problem solving plays a great and crucial
role in the process of studying calculus.
Note: The way to master calculus is to solve lots of calculus problems!

2
3.

Principles of problem solving

There are no exact rules for solving problems. However, it is possible to outline some general steps, or to give some
hints or principles that may be useful in the solution of certain problems.
The following steps and principles, which have been adapted from George Polyas book How to solve it, are in fact
just common sense made explicit.
Step 1:
Understand the problem. Read the problem and make sure that you understand what it is about. Ask
yourself the following questions: What is the unknown? What are the given quantities? What conditions are given?
In many cases it is useful to draw a given diagram and identify the given quantities in the diagram. It is usually
necessary to introduce suitable notation. In choosing symbols for the unknown quantities, we use letters such as
a, b, c . . . x and y, and it also helps to use abbreviations or symbols such as V (volume) and t (time).
Step 2:
Think of a plan. Try to find the connection between the data and the unknown. You may have to relate
a problem to others you have seen before. Have you seen the same problem in a slightly different form? Do you
know a similar/related problem? Do you know a theorem, rule, method or result that could be useful? Could you
reformulate the problem? Could you restate it still differently? Go back to your definitions.
If you cannot solve the proposed problem, try to solve some related problem first. Sometimes you will be able to
find a similar problem, while in other instances you may have to use components, sections, and methods of other
problems to solve a new one. Every problem you encounter becomes a potential method for solving other problems.
Step 3:
Carry out the plan. In order to carry out your plan, you have to check each step. Can you see clearly
that the step is correct? Can you prove that it is correct?
Step 4:
Look back. Examine the solution you have obtained. Can you check the result? Can you check the
argument? Can you derive the solution differently? Can you see it at a glance? Can you use the result, or the
method, for some other problem?

3
4.

MAT1512/1

Summary

Table 1 below is a summary of the mathematical ideas you need to master, with references to the relevant examples
in your textbook. Try to do as many review exercises as you can.

Table 1

5.

Topic(s)

Section in Smith and


Minton

I.

Polynomials and rational


functions

Examples 1.1 1.23

II.

Sketching graphs of different


functions

Examples 2.2 2.7

III.

Inverse functions

Examples 3.1 3.8

IV.

Trigonometric and inverse


trigonometric functions

Examples 4.1 4.10

V.

Exponential and logarithmic


functions

Examples 5.1 5.11

VI.

Transformation of functions

Examples 6.1 6.9

The way forward

The topics in the rest of the study guide are:


Chapter 1:
Chapter 2:
Chapter 3:
Chapter 4:

Limits and continuity


Differentiation of different types of functions
Integration
First-order differential equations, growth and decay, and partial derivatives.

CHAPTER 1
LIMITS AND CONTINUITY
1.

Background

The idea of a limit underlies the various branches of calculus. In fact, without limits, calculus simply would not
exist. Every single notion of calculus is a limit in one sense or another and the idea of a limit plays a fundamental
role in concepts such as instantaneous velocity, the slope of a curve, the length of a curve, the sum of infinite series,
etc. The role played by limits in both differential and integral calculus is crucial. In this chapter, three main ideas
will be introduced. They are: (a) the basic rules (definitions) of limits and their applications in evaluating limits
of algebraic and trigonometric functions; (b) the application of limits to continuity of functions; and (c) the use of
the Squeeze Theorem in determining certain limits. Begin by reading through the preview of this chapter on page
74 in Smith. You will notice that a mathematical model is used to predict how a function will behave. Graphs are
also used to represent function behaviour and will later be used to solve problems. This approach is used to solve
various calculus problems.
2.

Learning outcomes

At the end of this chapter, you should be able to


demonstrate an understanding of the concepts relating to limits and how these are applied in evaluating limits
of the form lim f (x), lim+ f (x), lim f (x), lim f (x) and lim f (x) , where c is a real number and f
xc

xc

xc

is an algebraic or trigonometric function

x+

use the limit definition of continuity


solve problems based on the Squeeze Theorem

5
3.

MAT1512/1

Prescribed reading

In Smith & Minton, Chapter 1, Sections 1.1, 1.2, 1.3, 1.4, 1.5 and appendix A, theorem A.1, theorem A.2, theorem
A.3, theorem A.4, theorem A.5, and theorem A.6.
Note: The way to master calculus is to solve a lot of calculus problems!
4

The limit

4.1

Introduction to the limit concept

Read carefully through section 1.1 in Smith. You will notice that we have two problems one dealing with finding
the slope of a curve at a specific point and the other one dealing with finding the length of a curve. In the first
problem, the closer the second point gets to a certain point, the closer the computed value gets to the actual value
we are looking for. This calculus problem involves a process called the limit. You can estimate the slope of the
curve using a sequence of approximations. The limit allows you to compute the slope exactly. Similarly, in the
second problem in trying to compute the distance along a curved path you would have to go through better
approximations successively until you reach the actual value. The actual arc length is obtained by using the limit.
The limit value is the height or the y-value of a function.
A left-hand limit is the y-value you obtain by approaching x from the left side.
A right-hand limit is the y-value you obtain by approaching x from the right side.
Let us use two functions to explore the limit concept. (These are the same functions as those used in section 1.2
x2 4
, is undefined at x = 2, but its behaviour can be
of your prescribed textbook.) The first function, f (x) =
x 2
examined in the area close to x = 2. From the value tables and the accompanying graph on page 80 in Smith, you
can see that the limit of f (x) as x approaches 2 from the left or right is 4.

4
x

f (x) =

1.9
1.99
1.999
1.9999

3.9
3.99
3.999
3.9999

x2 4
x 2

_2

lim f (x) = 4

f (x) =

2.1
2.01
2.001
2.0001

4.1
4.01
4.001
4.0001

x2 4
x 2

x
lim f (x) = 4

x2

x2+

This can be written mathematically as follows:


lim f (x) = 4.

x2

Consider another function g (x) =

x2 5
x2 5
or y =
.
x 2
x 2
y
10

x2 5
x 2

g (x) =

1.9
1.99
1.999
1.9999

13.9
103.99
1003.999
10,003.9999

x
5

10

_5

x2 5
x 2

y=

2.1
2.01
2.001
2.0001

-5.1
-95.99
-995.999
-9,995.9999

_ 10

lim g (x) = 10, 004

lim g (x) = 9, 996

x2

x2+

The limit of g(x) as x approaches 2 from the left, and the limit of g(x) as x approaches 2 from the right are not
equal. Mathematically, this can be written as follows: lim g (x) = lim+ g (x) and this statement means that
x2

x2

lim g (x) does not exist.

x2

4.2.
Definition of a limit: left- and right-hand limits
We say a limit exists if and only if the corresponding one-sided limits are equal. That is, if
lim f (x) = lim+ f (x) . In general, lim f (x) = L for some number L if and only if
xa

xa

xa

lim f (x) = lim+ f (x) = L .

xa

xa

Let us now consider some worked examples.


5.
Worked examples
Our collection of worked examples of the work covered in this chapter of Smith can be divided into eight sets (with
some overlap), namely:
I. Limits as x c, c R (cancellations)
II. Limits as x
III. Limits involving absolute values
IV. Left-hand and right-hand limits
V. Limits involving trigonometric functions
VI. The Squeeze Theorem
VII. The definition of a limit
VIII. Continuity

MAT1512/1

The table below (Table 2) shows which sections of Smith, and which worked examples in the study guide you must
consult.
Table 2
Topic(s)

Sections in Smith & Minton

Study guide
examples

I.

Limits as x c, c R

Sections 1.1, 1.2 and 1.3

112

II.

Limits as x

Section 1.5 (examples 5.6 5.12)

1327

III.

Limits involving absolute values

Section 1.2 (example 2.5)

2227

IV.

Left-hand and right-hand limits

Sections 1.2, 1.3 (example 3.9), 1.4


and 1.5 (examples 5.3 5.5 & 5.9)

2840

V.

Limits involving trigonometric


functions

Section 1.2 (example 2.4) and 1.3


(example 3.6)

4152

VI.

The Squeeze Theorem

Section 1.3 (example 3.8)

5357

VII.

The definition of a limit

Sections 1.6 and 1.7 (read only)

5863

VIII.

Continuity

Section 1.4 (examples 4.1 4.5)

6470

In the following sections, we present a number of worked examples based on the mathematical concepts and ideas
in each section of this chapter, together with appropriate teaching texts and references to Smith. The solutions to
the problems appear immediately after each set of examples. Unlike the exercises in Smith, the examples in the
study guide are not arranged in order of difficulty. Only attempt them once you have studied the relevant parts and
done some of the exercises in Smith.
I.

Limits as x c (c R)

Determine the following limits:


1.

x3 + 1
x1 x 2 + x + 2

2.

3.

x 2 2x + 1
x1 x 3 2x 2 + x

4.

lim

lim

lim

x3 x 2

x +3
+ 4x + 3

5x 3 + 8x 2
x0 3x 4 16x 2
lim

5.

x 2 2x 3
x1
x2 1

6.

x2 9
x3 x 3 27

7.

x4 1
x1 x + 1

8.

x 3 27
x3 x 2 3x

9.

lim

lim

lim

x1

x 1

x 2 + 24 5
1

11.

lim

x2

x 2 2x 3 2
x 2 2x 8

lim

lim

10. lim

h0

(x + h)2 + 1
h

x2 + 1

(x 2) 3 2
12. lim
x10
x 10

Solutions:
1.

x3 + 1
x1 x 2 + x + 2
(x + 1) x 2 x + 1
= lim
x1
(x + 1) (2 x)
x2 x + 1
= lim
x1
2x
3
= 3 = 1.
lim

x 2 2x + 1
x1 x 3 2x 2 + x
(x 1)2
= lim
x1 x (x 1)2
1
= lim = 1.
x1 x

3. = lim

2.

4.

x +3
+ 4x + 3
x +3
= lim
x3 (x + 3) (x + 1)
1
= lim
x3 x + 1
1
=
= 12 .
3 + 1
lim

x3 x 2

5x 3 + 8x 2
x0 3x 4 16x 2
x 2 (5x + 8)
= lim 2
x0 x 3x 2 16
5x + 8
= lim 2
x0 3x 16
lim

8
= = 16
= 12 .

5.

x 2 2x 3
x1
x2 1
(x + 1) (x 3)
= lim
x1 (x + 1) (x 1)
x 3
= lim
x1 x 1
4
=
= 2.
2
lim

6.

x2 9
x3 x 3 27
(x 3) (x + 3)
= lim
x3 (x 3) x 2 + 3x + 9
x +3
= lim 2
x3 x + 3x + 9
6
= 29 .
= 27
lim

9
x4 1
x1 x + 1
x2 1 x2 + 1
= lim
x1
x +1
(x 1) (x + 1) x 2 + 1
= lim
x1
x +1
= lim (x 1) x 2 + 1

7.

MAT1512/1
x 3 27
x3 x 2 3x
(x 3) x 2 + 3x + 9
= lim
x3
x (x 3)
x 2 + 3x + 9
= lim
x3
x
= 9.

lim

8. = lim

x1

= (1 1) (1)2 + 1
= 4.
In the following two solutions we make use of the identity (i) and rationalising (ii):
(i) A2 B 2 = (A B) (A + B)
(ii) Rationalising the denominator, eg

1
x
x
1
= =
x
x
x
x
(A B)(A + B) = A2 B 2 .

9.
lim

x1

x 1

x 2 + 24 5

= lim

x1

= lim

x1

= lim

x1

= lim

x1

10.
lim

h0

= lim

h0

(x + h)2 + 1
h

(x + h)2 + 1
h

x 2 + 24 25

x 2 + 24 + 5
(x 1)
x2 1

x 2 + 24 + 5
(x 1)

x2

(x 1) (x + 1)
+ 24 + 5
= 5.
x +1

x2 + 1

(x + h)2 + 1 +

(x + h)2 + 1 + x 2 + 1
x2 + 1

(x + h)2 + 1 x 2 + 1

h0
h
(x + h)2 + 1 + x 2 + 1

= lim

x 2 + 2xh + h 2 + 1 x 2 1

h0
h
(x + h)2 + 1 + x 2 + 1

= lim
= lim

h0

2x + h

(x + h)2 + 1 +

x 2 + 24 + 5

x 2 + 24 5
x 2 + 24 + 5

x 2 + 24 + 5
(x 1)

x1

= lim

x 1

x2 + 1

x
x2 + 1

x2 + 1

10
In the following two solutions we make use of the identity
(A B)(A2 + AB + B 2 ) = A3 B 3 .
11.
lim

x2

1
3

1
3

x 2x 2
x 2x 2 x 2x
= lim 2

2
x2
x 2x 8
x 2x 8
x 2 2x

2
3
2
3

=
=
=

lim

x2

x 2 2x 8

lim

x 2x 8

x 2 2x

2
3

2
3

x2

1
3

x 2 2x + 2 x 2 2x
1
1
= .
4+4+4
12

1
3

+ 2 x 2 2x

+ 2 x 2x
+2

x2

2x

1
3
1
3

+4
+4

+4

+4

(Here we put A = (x 2 2x) 3 and B = 2, and multiplied both numerator and denominator by A2 + AB + B 2 .)
12.
1

(x 2) 3 2
lim
x10
x 10
1

x10

x10

=
=
II.

lim

lim

lim

x10

(x 2) 3 2 (x 2) 3 + 2 (x 2) 3 + 4

2
1
x 10
(x 2) 3 + 2 (x 2) 3 + 4
(x 2) 8
2

(x 10) (x 2) 3 + 2 (x 2) 3 + 4
1

2
3

(x 2) + 2 (x 2) 3 + 4
1
1
= .
4+4+4
12

Limits as x

In this case, if you are required to find the limit of a function which is a polynomial divided by another polynomial,
find the highest power of the variable in the denominator and divide each term in both numerator and denominator
by that power.
Note:
lim 1
x x
Example:
6x 2 1
x 2x 2 + 7x

Find the following limit: lim

=0

11

MAT1512/1

Solution:
6x 2 1
lim
x 2x 2 + 7x

=
=
=
=

6x 2
2
lim 2xx 2
x
x2

lim

2+

1
x2
7x
x2

Notice that the highest power in the


denominator is x 2 .

1
x2
7
x2

Divide each term in both numerator


and denominator by x 2 .

60
2+0
6
= 3.
2

Now determine the following limits:


13.

lim

x4 + x3
x 12x 3 + 128

14.

7x 3 + 1
x x 2 + x

15.

7x 3 + 1
x x 2 + x

16.

9x 4 + x
x 2x 4 + 5x 2 x + 6

17.

19.

21.

lim

lim

x 2 + 4x 2x
2x

lim

lim

x3 + x2

x 2 + 2x + x
1
3

18.

20.

lim

lim

lim

lim

x2

4x + 8

+ 7x 2x
1

x 2 + 5x + x

Solutions:

13.
x4 + x3
x 12x 3 + 128
x +1
= lim
x 12 + 128/x 3
= .
lim

14.
7x 3 + 1
x x 2 + x
7x + 1/x 2
= lim
x 1 + 1/x
= .
lim

12
15.
7x 3 + 1
x x 2 + x
7x + x12
= lim
x 1 + 1
x
= .
lim

16.
9x 4 + x
x 2x 4 + 5x 2 x + 6
9 + 1/x 3
= lim
x 2 + 5/x 2 1/x 3 + 6/x 4
lim

lim

9 + 1/x 3

2 + 5/x 2 1/x 3 + 6/x 4


9+0
2+00+0
9
.
2

=
=
17.

x 2 + 4x 2x
x
2x
x 2 (1 + 4/x) 2x
lim
x
2x

|x| 1 + 4/x 2x
lim
x
2x

x 1 + 4/x 2x
lim
(|x| = x since x < 0)
x
2x

x 1 + 4/x + 2
lim
x
2x

1 + 4/x + 2
3
= .
lim
x
2
2
lim

=
=
=
=
=

18.
lim

=
=
=
=
=

lim

4x + 8

x 2 + 7x 2x
4x + 8

x 2 (1 + (7/x)) 2x

4x + 8
( x 2 = |x|)

lim
x |x| 1 + (7/x) 2x
4x + 8
(|x| = x since x < 0)
lim

x x 1 + (7/x) 2x
4 + (8/x)
lim

x 1 + (7/x) 2
4
4+0
= .

3
1+02
x

13

MAT1512/1

19.
x 2 + 2x + x

x 2 + 2x + x
x 2 + 2x x

lim
x
x 2 + 2x x
x 2 + 2x x 2
lim
x
x 2 + 2x x
2x
lim

x |x| 1 + 2/x x
2x
(|x| = x since x < 0)
lim

x x 1 + 2/x x
2
= 1.
lim

x 1 + 2/x 1
lim

=
=
=
=
=
20.

lim

=
=
=
=
=
=
=

lim

1
x2

+ 5x + x

x 2 + 5x + x

x 2 + 5x x
x 2 + 5x x

x 2 + 5x x
x
5x
2
x (1 + 5/x) x
lim
x
5x

|x| 1 + 5/x x
x 2 = |x|
lim
x
5x

x 1 + 5/x x
lim
(|x| = x since x < 0)
x
5x

1 + 5/x 1
lim
x
5

1+01
2
= .
5
5
lim

21.
x

x3 + x2

lim

x3 + x2

lim

=
=
=

1
3

1
3

lim

x3 + x2

x3 + x2

lim

x3 + x2

2
3

+ x x3 + x2
x2

+ x x3 + x2

+ x x3 + x2

x3 + x2 x3
2
3

2
3

1
3

1
3

+ x2
1

x 2 (1 + 1/x) 3 + x 2 (1 + 1/x) 3 + x 2
1
1
= lim
= .
2
1
x
3
(1 + 1/x) 3 + (1 + 1/x) 3 + 1

+ x2

1
3

+ x2

14
III.

Limits involving absolute values

Remember from high school that


x if x 0
x if x < 0

|x| =

The definition of absolute value gives a function in two sections (parts) with corresponding intervals,
ie if y = |x| then
x if x 0
y=
x if x < 0

Another example:
If

3 2x, if 3 2x 0 3 2x
(3 2x) if 3 2x < 0 3 < 2x

f (x) = |3 2x| =
Thus,

f (x) =
Let us look at one example:

3 2x, if x
3 + 2x, if x >

Find the following limit:


lim

Solution:
x 5
x |3 2x|
lim

=
=
=
=

x 5
= lim
x 3 + 2x
x
lim

lim

x 3
x

10
0 + 2
1
2

Note: Since we have x , x >

3
2

5
x

x
x
3
x

3
2
3
2

x x 32
< x x > 32 .

3
2
3
2

x 5
|3 2x|

5
x
2x
x

|3 2x| = (3 2x) since x >

3
2

+2

and this is why we have |3 2x| = 3 + 2x in our example above.

15
Now determine the rest of the limits below.
22.

24.

26.

3x + 1
x |2 x|
lim

lim

23.

x
|x|

25.

|x + 1|
x 2x + 3
lim

27.

|x|
x |x| + 1
lim

lim

|x + 1|
x +1

2 + |2x + 1|
x
2x + 1
lim

Solutions:
22.
|2 x| =
=

2x
if 2 x 0
(2 x) if 2 x < 0
2 x if x 2
x 2 if x > 2

Hence
3x + 1
|2 x|
3x + 1
= lim
( |2 x| since x > 2)
x x 2
3 + 1/x
= lim
x 1 2/x
3+0
= 3.
=
10
lim

23.

|x|
x |x| + 1
x
= lim
(|x| = x since x < 0)
x x + 1
1
= lim
= 1.
x 1 + 1
x
lim

24.
x
x |x|
x
(|x| = x since x < 0)
= lim
x x
= lim (1) = 1.
lim

25.
|x + 1|
x +1
(x + 1)
(|x + 1| = (x + 1) since x < 1)
= lim
x
x +1
= lim (1) = 1.
lim

MAT1512/1

16
26.
|x + 1|
2x + 3
(x + 1)
(since x < 1)
= lim
x 2x + 3
1 1/x
1
= lim
= .
x 2 + 3/x
2
lim

27.
2 + |2x + 1|
2x + 1
1
2 (2x + 1)
(|2x + 1| = (2x + 1) since x < )
= lim
x
2x + 1
2
1 2x
= lim
x 2x + 1
1
2
= lim x 1 = 1.
x 2 +
x
lim

IV.

Left-hand and right-hand limits

A lefthand limit is the y-value you obtain by approaching x from the left side.
A righthand limit is the y-value you obtain by approaching x from the right side.

Let us try out one example:


Suppose

Evaluate the following limits:


28. lim h (x)
x0

29. lim h (x)


x0+

30. lim h (x)


x2

31. lim+ h (x)


x2

32. lim h (x)


x2

Solutions:
28. lim h (x) = lim x = 0.
x0

x0

x if x < 0
h (x) =
x 2 if 0 < x 2

8 x if x > 2.

17

MAT1512/1

29. lim h (x) = lim+ x 2 = 02 = 0.


x0+

x0

30. lim h (x) = lim x 2 = 22 = 4.


x2

x2

31. lim+ h (x) = lim+ (8 x) = 8 2 = 6.


x2

x2

32. lim h (x) does not exist, since lim h (x) = lim+ h (x)
x2

x2

x2

Now determine the following limits:


33. lim
x2

1
.
x 2

34. lim f (x), where


x2

35. lim

1
.
|x|

36. lim

x
.
|x|

x0

x0

x + 2 if x > 2
f (x) =
if x < 2
x2

0
if x = 2.

37. Let
f (x) =

2
if x > 1
2
1 x if x < 1.

(a) Draw the graph of f.


(b) Determine lim f (x) and lim+ f (x).
x1

x1

(c) Does lim f (x) exist? Give a reason for your answer.
x1

38. Let

Determine lim+ f (x) ,


x2

39. Determine lim


x0

x + 1 if x > 2
f (x) =
0
if x = 2 or x = 0

1
if x < 2 and x = 0.
x

lim f (x) ,

x2

lim f (x),

x2

lim f (x) and lim f (x), if these limits exist.

x
.
tan x

40. Let

3 x if x < 2
f (x) =
2
if x = 2

x
if x > 2.
2

Find lim+ f (x) and lim f (x). Does lim f (x) exist? If so, what is it?
x2

x2

x2

18
Solutions:
1
x2 x 2
= .

33. lim

34. lim f (x) = lim x 2 (since x < 2)


x2

x2

= 4.
35. lim
x0

1
|x|

= lim
x0

=.

1
(|x| = x since x < 0)
x

36. If x tends to 0 , then x is negative. According to the definition of the absolute value, we then have that
|x| = x.
Consequently,
x
lim
x0 |x|
x
= lim
= lim (1) = 1.
x0 x
x0
37.

(a)

y
2
y = f (x)
1

x
0

(b) lim f (x) = lim (1 x 2 ) = 1 1 = 0.


x1

x1

x1

x1

lim+ f (x) = lim+ (2) = 2.

19

MAT1512/1

(c) lim f (x) does not exist, because


x1

lim f (x) = lim f (x) .

x1+

x1

38. lim f (x) = lim+ (x 2 + 1) = 5.


x2+

x2

1
1
= .
x2
x2
x
2
lim f (x) does not exist, since lim+ f (x) = lim f (x).
lim f (x) = lim

x2

x2

x2

lim f (x) = lim (x + 1) = .

1
x

lim f (x) = lim

39. lim
x0

x
tan x

= lim
x0

=
=

1
lim

x0

1
1

= lim
x0

x
sin x
sin x
x

x
sin x

= 0.

cos x

lim cos x
x0

lim cos x
x0

1 = 1.

40. lim+ f (x) = lim+


x2

x2

x
2
= =1
2
2

while
lim f (x) = lim (3 x) = 3 2 = 1.

x2

x2

lim f (x) exists, since lim+ f (x) = lim f (x).

x2

x2

x2

We have lim f (x) = 1.


x2

V.
Limits involving trigonometric functions
In order to compute the derivatives of the trigonometric functions (for example, by using the first principle of
differentiation), we first have to know how to evaluate the limits of trigonometric functions.
Below are some of the theorems you need to know:
(i) lim sin x = 0,
x0

(ii) lim cos x = 1,


x0

sin x
x
= lim
= 1,
x0 x
x0 sin x

(iii) lim
(iv) lim

x0

cos x 1
1 cos x
= lim
= 0.
x0
x
x

20
Examples:
Find the limit of the following:

sin (5)
0
3
lim

Solution:
In order to apply theorem (iii) above, we first have to re-write the function as follows:
sin 5 5
5 sin 5
5
sin 5
=
. =
=
3
3 5
3.5
3

sin 5
5

Notice that as 0 we have 5 0. So by theorem (iii) above, with x = 5,


we have lim

sin 5
sin (5)
= lim
= 1.
5 0
5
5

Therefore,
sin 5
0 3
lim

5 sin (5)
0 3
5
5
sin 5
=
lim
0
3
5
5
=
.1
3
5
.
=
3
= lim

Also find the limit of


lim x cot x.

x0

Solution:
lim x cot x = lim x

x0

x0

= lim

cos x
sin x

cos x

x0 sin x
x

lim cos x

=
=

x0

lim

x0

sin x
x

1
= 1.
1

21

MAT1512/1

Also, determine the following limits:


2
0 tan

42. lim

x + sin x
x0
x

44. lim

sin (5)
0 sin (2)

46. lim (tan (2x) csc (4x))

sin (2 10)
5 sin ( 5)

48. lim

sin (5 )
0
2

41. lim

tan (2t)
t0
t

43. lim

45. lim

x0

1 cos
0

47. lim
49.

51.

lim

x sin 1x

50.

lim

x 2 sin 1x

52. lim

lim x sin 1x

1 cos
0 1 cos (3)

Solutions:

41.

2
2
= lim sincos
0 tan
0

cos
= lim
0 sin

lim ( cos )
= lim
0 sin 0
= lim sin lim lim cos
lim

42.

sin (5)
2
5 sin (5)
= lim
0
2.5
5
1 = 5/2.
=
2
lim

= 1 0 1 = 0.

43.

x + sin x
x0
x
sin x
= lim 1 +
x0
x
sin x
= lim 1 + lim
x0
x0 x
lim

44.

= 1 + 1 = 2.
45.

sin (5)
sin (2)
2
sin (5 ) 5
= lim

0
5
2 sin (2)
sin (5) 5
2
= lim
lim
0
5
2 0 sin (2 )
5
5
= 1 1= .
2
2
lim

46.

tan (2t)
t0
t
sin (2t)
= lim
t0 t cos (2t)
2
sin (2t)
= lim

t0
2t
cos (2t)
2
= 1 = 2.
1
lim

lim (tan (2x) csc (4x))

x0

sin (2x)
x0 cos (2x) sin (4x)
1
sin (2x)
= lim

x0 cos (2x) 2 sin (2x) cos (2x)


1
1
1
= .
= lim

2
x0 2 cos (2x)
2

= lim

22
47. Put x = 5. Then 2 10 = 2x and x 0 5. Now,
sin (2 10)
5 sin ( 5)
lim

sin (2x)
x0 sin x
x
sin (2x) 2x
= lim

x0
2x
x sin x
= 1 2 1 = 2.
= lim

48.
1 cos
(1 cos ) (1 + cos )
= lim
0
0

(1 + cos )
2
1 cos
lim
0 (1 + cos )
sin2
lim
0 (1 + cos )
sin
sin
lim
lim
0
0 1 + cos
1.0 = 0.
lim

=
=
=
=
49. Put =

1
1
. Then x = and x 0 . Hence,
x

lim

x sin

1
x

sin

lim

= 1.
50.
lim

x sin

lim+

sin

x0

1
x
(where =

= 1.
51. Put = 1x . Then x 0 . Thus
lim

lim

= .

1
x
sin 1

x 2 sin

1
)
x

23

MAT1512/1

52.
1 cos
0 1 cos (3)
1 + cos 1 + cos (3)
1 cos

= lim

0 1 cos (3) 1 + cos 1 + cos (3)


lim

= lim

1 cos2 (1 + cos (3 ))

1 cos2 (3) (1 + cos )

sin2 (1 + cos (3))


0 sin2 (3) (1 + cos )

= lim

sin

= lim

= 12
VI.

2
3
2
sin (3 )
9

1 2 2
1
1 = .
9
2
9

1 + cos (3)
1 + cos

The Squeeze Theorem

Note: The Squeeze Theorem is also known as the Sandwich Theorem or Pinching Theorem.
The Squeeze Theorem is used to evaluate the limit values of a complicated function, or the limit values of an
unknown function. The Squeeze Theorem is stated as follows:If f (x) g (x) h (x) for all x in an open interval that contains a (except possibly at a)
and lim f (x) = lim h (x) = L, then lim g (x) = L.
xa

xa

xa

The Sandwich Theorem is illustrated by the sketches below.


Imagine you have a sandwich as shown in the following figure:
The filling is taken as a
complicated function,
y = g(x )

y = h(x )
y = g(x )
y = f (x )

The first slice is


taken as a
smaller function,
y = f (x )

The second slice is


taken as a bigger
function,
y = h(x )

The theorem says that if g(x) is squeezed or pinched between f (x) and h(x) near a, and if f and h have the same
limit value L at a, then g is forced to have the same limit value L at a.
Suppose you have to find the following limit:
lim x 2 sin

x0

1
x

24
This is a complicated function, in other words, it is difficult to determine the limit of the function. In such a case we
can use the Squeeze (Sandwich) Theorem.

Squeeze Theorem
If we have f (x) g (x) h (x) and for some value L,
lim f (x) = L and lim h (x) = L,

xc

xc

then, by the Sandwich (Squeeze) Theorem


lim g (x) = L

xc

Now let us find the limit of


lim x 2 sin x1 .

x0

(This is an example of a complicated function.)

Solution:
To use the Squeeze Theorem, we first have to find functions f and h such that
f (x) x 2 sin

1
h (x)
x

where
lim f (x) = lim h (x) .

x0

Recall that
1 sin

x0

1
x

1 for all x = 0.

Now we multiply by x 2 (note that for x = 0, x 2 > 0).

This means that the inequality signs will not change in direction, and we get
x 2 (1) x 2 . sin

1
x

x 2 .1, which means that x 2 x 2 . sin

Then lim x 2 = 0 and lim x 2 = 0.


x0

1
x

x 2 for all x = 0.

x0

So from the Sandwhich Theorem, it follows that lim x 2 . sin


x0

1
x

= 0.

Work through examples 3.7 and 3.8 on pages 92 to 94 in your prescribed textbook (Smith & Minton).
After that, go through the given worked examples below.

53. Find lim f (x) if


x

2x 2 + 5
2x 2
.
<
f
(x)
<
x2 + 1
x2

25
54. Suppose that

MAT1512/1

x2
x sin x
<
<1
6
2 2 cos x

1
for x close to zero. Determine

lim

x0

55. Use the Squeeze Theorem to determine

x sin x
.
2 2 cos x

sin x
.
x x
lim

56. Use the Squeeze Theorem to determine


lim ( x /x) and

lim ( x /x) .

57. Use the Squeeze Theorem to determine

1 cos x 2
.
x 1 + x 3
lim

Solutions:
2x 2
2
= 2 and
= lim
2
x x + 1
x 1 + 1/x 2

53. lim

2x 2 + 5
2 + 5/x 2
=
lim
= 2.
x
x
x2
1
lim

By the Squeeze Theorem, it follows that lim f (x) = 2.


x

lim 1

54.

x0

x2
6

=1

0
= 1,
6

while
lim 1 = 1;

x0

so, by the Squeeze Theorem

x sin x
= 1.
x0 2 2 cos x
lim

55. For all x it holds that

1 sin x 1.
If x < 0, we can divide by x to obtain

1
1
= 0, and lim
x x
x
x

Now lim

sin x
1
1

.
x
x
x
1
= 0. It follows by the Squeeze Theorem that
x x

= lim

sin x
= 0.
x x
lim

26
56. For all x it holds that
x 1 < x x.
If x > 0 then

(*)

x 1
< x /x 1.
x

Now

1
x 1
= lim
x
x
x
1
lim

1
x

= 1;

while lim 1 = 1. From the Squeeze Theorem it follows that


x

lim ( x /x) = 1.

If x < 0, then we can divide () by x to obtain


x 1
> x /x 1.
x
Similarly, as above, it follows that
lim ( x /x) = 1.

57. For all x it holds that


1 cos x 2 1
Add 1:
0 1 cos x 2 2
For x > 0 (because x ), we divide by 1 + x 3 > 0 to obtain
1 cos x 2
2
0

.
3
3
1+x
1+x
1 + x3
2
= 0. It follows by the Squeeze Theorem that
x 1 + x 3

Now lim 0 = 0 and lim


x

1 cos x 2
= 0.
x 1 + x 3
lim

VII.
The definition of a limit (read only)
This section is not for examination purposes; therefore you should not spend too much time on it. However, those
of you who intend to continue with pure mathematics should be familiar with the contents of this section.

58. Determine > 0 so that


0< x

1
1
1
<
1 <
.
4
4x
20

59. Prove from the definition that lim (7x 1) = 13.


x2

27

MAT1512/1

60. Prove from the definition that lim f (x) = 10, where f (x) = 3x + 7.
x1

x2 4
= 4.
x2 x 2

61. Prove from the definition that lim

x2 1
= 2.
x1 x 1

62. Prove from the definition that lim

x2 9
= 6.
x3 x + 3

63. Prove from the definition that lim


Solutions:
58.

1
1
1 <
4x
20
1
1
1
<
1<
20
4x
20
1
1
1
<
1+1<
+1
1
20
4x
20
1
21
19
<
<

20
4x
20
20
20

> 4x >
19
21
5
5

<x<
21
19
5
1
1
5
1

<x <

21 4
4
19 4
1
1
1
.
<x <
84
4
76
1
If we choose = , we should have that
84
0< x

1
1
1
<
1 <
.
4
4x
20

59. According to the definition of a limit, let f (x) = 7x 1, xo = 2 and L = 13. Given
> 0 so that for all x
0 < |x 2| < | f (x) 13| < .
Now
| f (x) 13| = |7x 1 13|
= |7x 14|

= 7 |x 2|
So let = . Then
7

0 < |x 2| < | f (x) 13| = 7 |x 2| (from above)


< 7

= 7

= .

> 0, we need to find

28
60. We have that
| f (x) 10| = |3x + 7 10| = |3x 3| = |3(x + 1)|
= 3|x + 1|

= 3 |x (1)| .

If

> 0 is given, put = . Then


3
0 < |x (1)| < | f (x) 10| = 3 |x (1)|
< 3 = 3

= .

This shows that


lim f (x) = 10.

x1

61. Let

> 0 be given. We have that


x2 4
4
x 2
(x + 2) (x 2)
=
4
x 2
= |x + 2 4| (if x = 2)

= |x 2|.

Put = . Then, if 0 < |x 2| < , it follows that x = 2, and so

x2 4
4 = |x 2| < = .
x 2

62. Let

> 0 be given. Then


x2 1
2
x 1

(x 1) (x + 1)
2
x 1
= |x + 1 2| (if x = 1)
=

= |x 1|.
Put = . Then
Hence,

0 < |x 1| <

x2 1
2 < .
x 1

x2 1
= 2.
x1 x 1
lim

63. In the definition of a limit, we put f (x) = (x 2 9)/(x + 3), xo = 3 and L = 6. Let > 0 be given. Then
x2 9
(6)
x +3

(x + 3) (x 3)
+6
x +3
= |x 3 + 6| (if x = 3)
=

= |x + 3|

= |x (3)|.

29
Let = . If

MAT1512/1

0 < |x (3)| <

then x = 3, and then it follows from the above that


x2 9
(6) = |x (3)| < .
x +3
Hence,

x2 9
= 6.
x3 x + 3
lim

VIII.

Continuity

In the previous topics, notice that the limit of a function as x approaches a can sometimes be found by just calculating
the value of the function (y) at x = a. The functions with this property are said to be continuous at a. A continuous
process is one that takes place gradually, without interruption or abrupt changes.
Definition 1
A function f is continuous at a number a if lim f (x) = f (a) .
xa

Definition 2
A function f is continuous from the right at a number a if lim+ f (x) = f (a)
xa

and f is continuous from the left at a number a if lim f (x) = f (a) .


xa

If f is not continuous at a, we say that f is discontinuous at a, or that f has a discontinuity at a. When


we consider the two definitions above, we notice that three things (or conditions) are required for f to be
continuous at a.
A function f is continuous at x = a when the following conditions are satisfied:
(i) if f (a) is defined (that is, a is in the domain of f )
(ii) if lim f (x) exists [that is, f must be defined in an open interval that contains a] and lim f (x) = lim+ f (x)
xa

xa

xa

(iii) if lim f (x) = f (a).


xa

If one or more of the three conditions above are not satisfied, then f is said to be discontinuous at x = a. [Try to
think of the process of finding a limit as making a prediction as to what the y-value will be when x finally gets to
a.]

30
In figures (a), (b), (c) and (d) below, three main types of discontinuities removable, jump and non-removable
discontinuities are shown. Are you able to explain why the function f is discontinuous at x = a in all four
diagrams?
Figure (a)

Figure (b)

[REMOVABLE]

[REMOVABLE]
f ( a) is undefined _
f ( x) exists,
but lim
x a
the graph has a
hole at x = a

f ( a)

lima f ( x) exists
x
and f (a ) is
defined but
lim
f ( x) = f ( a) ;
x a

f ( a)

Figure (c)

the graph has a


hole at x = a

Figure (d)

[JUMP]

[NONREMOVABLE]

The limit does not


exist, but
f ( a) is defined, the
graph has a jump at
x x =a

The limit does not


exist, the function value
or yvalue expands and
extends to infinity at
x = a.

Let us now work through some examples.


Example 1:
(a) Determine if the function f (x) =
and explain why.
(b) Sketch the graph of the function.

1 x,
x 2x,
2

if
if

x 2
x >2

is continuous or discontinuous at a = 2,

31

MAT1512/1

Solution:
(a)
lim f (x) =

x2

lim f (x) =

x2+

x2

x2+

lim (1 x) = 1 2 = 1
lim x 2 2x = (2)2 2 (2) = 4 4 = 0

Since
lim f (x) = lim+ f (x)

x2

x2

lim f (x) does not exist [that is, condition (ii) has not been met].

x2

Therefore the function is discontinuous at x = 2.


(b) The key points on the graph are:
(i) the vertical intercept(s) (where x = 0)
(ii) the horizontal intercept(s) (where f (x) = 0)
(iii) the turning points

y
3
f ( x) = x

2_

2x

f ( x) = 1 _ x

_2

_1

x
1

_1

_2

f (x) = 1 x is a straight line with intercepts at (0, 1) and (1, 0) for the interval where x 2.

f (x) = x 2 2x is a curve with intercepts where x(x 2) = 0 that is (0, 0) and (2, 0) are the x-intercepts,
but they do not fall in the interval x > 2.

32
Example 2:
If
f (x) =

x 2 x2
x2

if x = 2
1, if x = 2,

determine if the function f (x) is continuous at x = 2.


Solution:
For the lim f (x) to exist, we must first find lim f (x) and lim+ f (x) . Since
x2

x2

x2 x 2
x2
x2
x 2
(x 2) (x + 1)
= lim
= lim (x + 1) = 3,
x2
x2
x 2

lim f (x) =

x2

x2

lim+ f (x) = lim

the limit exists.


But f (2) = 1 is also defined and lim f (x) = f (2) .
x2
So f (x) is not continuous at x = 2 (this is the removable type of discontinuity).
Example 3:
Find the value of the constant c that would make the function f continuous at x = 3, where:
f (x) =

cx + 1, if x 3
cx 2 1, if x > 3.

Solution:
We determine the left- and right-hand limits as follows:
lim f (x) = lim (cx + 1) = 3c + 1

x3

x3

and
lim f (x) = lim+ cx 2 1 = 9c 1.

x3+

x3

The condition for f to be continuous at x = 3 is that lim f (x) = lim+ f (x) ,


x3

that is

3c + 1
9c 3c
6c
c

x3

=
=
=
=

9c 1
1+1
2
1
.
3

33
Thus f (x) is continuous at x = 3, for

MAT1512/1

1
c= .
3

You can now work through the last examples of this chapter.
64. Let
f (x) =

x
if x 1
2
(x 1) if x > 1.

(a) Draw the graph of f (x) .


(b) Determine lim+ f (x) and lim f (x). Does lim f (x) exist? Give a reason for your answer.
x1

x1

x1

(c) Is f (x) continuous at x = 1? Give a reason for your answer.

65. Let
f (x) =

4 x 2 if x > 1
if x < 1.
3x 2

(a) Draw the graph of f (x) .


(b) Calculate lim+ f (x) and lim f (x).
x1

x1

(c) What value, if any, must be assigned to f (1) to make f (x) continuous at x = 1? Give reasons for your
answer.

66.

(a) At which points (if any) is the function


f (x) =

x2

x +a
+ (a + b) x + ab

x2

x +a
+ (a + b) x + ab

discontinuous? (a and b are constants.)


(b) At which points (if any) can
f (x) =

be made continuous by assigning a certain value to f (x), and what is the value?
67. Let
g(x) =

x 2 +2x15
x3

if x = 3
if x = 3.

What value, if any, must k be for g to be continuous at x = 3? Give reasons for your answer.
68. Let
g(x) =

bx 2 if x 12
x 3 if x < 12 .

What value must b be for g to be continuous at x = 12 ? Give reasons for your answer.

34
69. Let

x 3 + 4 if x 0
g(x) =
b+1

if x > 0.
x +2

What value must b be for g to be continuous at x = 0? Give reasons for your answer.
70. Let
g(x) =

x 2
if x 2
x + 1 if x < 2.

What value must be for g to be continuous at x = 2? Give reasons for your answer.
Solutions:

64.

(a)
y

y = f (x)
1

x
1

(b) (i) lim+ f (x) = lim+ (x 1)2 = 0.


x1

x1

(ii) lim f (x) = lim (x) = 1.


x1

x1

(iii) lim f (x) does not exist, because lim+ f (x) = lim f (x).
x1

x1

x1

(c) The function f (x) is thus not continuous at x = 1, because lim f (x) does not exist.
x1

35
65.

MAT1512/1

(a)
y
y = f (x)

x
0

(b) lim+ f (x) = lim+ (4 x 2 ) = 4 1 = 3;


x1

x1

lim f (x) = lim (3x 2 ) = 3.

x1

x1

(c) Let f (1) = 3, since lim f (x) = 3.


x1

66.

x +a
+ (a + b) x + ab
x +a
=
.
(x + a) (x + b)

(a) f (x) =

x2

f (x) is discontinuous where


(x + a)(x + b) = 0
that is at x = a or x = b.
(b) The case a = b
We calculate the limits of f (x) when x tends to a and to b.
x +a
xb (x + a) (x + b)
1
= lim +
xb x + b
= .
lim +

36
The discontinuity at x = b cannot be removed. Now,
x +a
xa (x + a) (x + b)
1
= lim
xa x + b
1
=
.
ba
lim

The discontinuity at x = a can be removed. We can make f (x) continuous at x = a by putting


f (a) =

1
.
ba

The case a = b
In this case there is a non-removable discontinuity at x = b.
Note: Removable discontinuity is when a discontinuity can be removed by redefining the function at that
point. In the case of a nonremovable discontinuity, the limits do not exist, so there is no way to redefine the
function at that point of discontinuity. See pages 99-100 of Smith.
x 2 + 2x 15
x3
x 3
(x 3) (x + 5)
= lim
x3
x 3
= lim (x + 5)

67. lim

x3

= 8.
From the definition of continuity it follows that k must equal to 8 to make g continuous at x = 3.
1
1
). For this limit to exist
, lim g(x) must exist (and be equal to g
2 x 12
2
lim+ g(x) and lim g(x) must exist and be equal. Now

68. For g (x) to be continuous at x =


x 12

x 12

lim+ g(x) = lim+ (bx 2 ) =

b
4

lim g(x) = lim (x 3 ) =

1
.
8

x 12

x 12

and
x 12

Thus we must have that

or

x 12

1
b
=
4
8
1
b= .
2

37

MAT1512/1

69. The function g (x) will be continuous at x = 0 if


g (0) = lim g (x) .
x0

Also, lim g(x) will exist provided that lim g(x) and lim+ g(x) exist and are equal. Hence we must determine
x0

x0

x0

the latter two limits, set them equal, and solve for b. Now

lim g(x) = lim (x 3 + 4) = 0 + 4 = 4,

x0

x0

while

b+1
x +2

lim+ g(x) = lim+

x0

x0

Let

b+1
b+1
=
.
0+2
2

b+1
= 4.
2

Then
b = 7.
Hence if b = 7, then g (x) is continuous at x = 0.
70. If g (x) is to be continuous at x = 2, then it must hold that
g(2) = lim g(x),
x2

that is
4 = lim+ g(x).
x2

...()

Now lim g(x) will exist provided lim+ g(x) and lim g(x) exist, and
x2

x2

x2

lim+ g(x) = lim g(x).

x2

x2

We have that
lim g(x) = lim+ (x 2 ) = 4,

x2+

x2

while
lim g(x) = lim (x + 1) = 3.

x2

x2

Hence both () and () will be satisfied if 4 = 3. Thus if = 34 , then g (x) is continuous at x = 2.

...()

38
Key points
You should now be comfortable with the notion of the limit of a function and its use. This includes one-sided limits,
infinite limits (limits resulting in infinity) and limits at infinity (limits of functions as x approaches infinity).
At this stage you should be able to
evaluate limits of the form lim f (x), lim f (x) , lim+ f (x) and lim f (x) , where c is a real number and
xc

xc

f (x) is an algebraic or trigonometric function

xc

provide a formal definition of the limit, using the correct mathematical notation
apply the laws governing limits
determine and evaluate the limits of sums, products, quotients and compositions of functions
evaluate the limits of functions analytically, graphically and numerically
use the limit definition of continuity to determine whether a function is continuous or discontinuous at a point
use the Squeeze Theorem to determine certain limits

At the end of each exercise set in Smith & Minton, there is a section entitled Exploratory exercises. Try the
questions in these sections they may help you order your thoughts. Also try the questions in the section For your
review at the end of the chapter.

39

MAT1512/1

CHAPTER 2
DIFFERENTIATION OF DIFFERENT
TYPES OF FUNCTIONS
1.

Background

Differentiation is widely applied in engineering, chemistry, physics, biology and many other disciplines in science
and technology. It is basically a concept that deals with rate of change. Among other things, it is used to define the
slopes of curves, to calculate the velocities and accelerations of moving objects, to find the firing angle that gives a
cannon its greatest range, and to predict the times when planets will be closest to each other or farthest apart. In this
chapter we begin the study of differential calculus. The central concept of differential calculus is the derivative.
After learning how to calculate derivatives, you will use them to solve problems involving rates of change. This
chapter is devoted to (a) the definition of the derivative, or differentiation from first principles; (b) the rules used
to find derivatives of algebraic and trigonometric functions; (c) problems involving tangent and normal lines; (d)
techniques such as logarithmic differentiation and implicit differentiation; and (e) the application of the Mean Value
Theorem and Rolles Theorem.
2.

Learning outcomes

At the end of this chapter, you should be able to

demonstrate an understanding of the first principles of differentiation


apply basic differential formulas such as the power rule, product rule, quotient rule, chain rule, and combinations of these rules to differentiate a variety of algebraic and trigonometric functions
compute the derivatives of trigonometric functions and inverse trigonometric functions, and of exponential
and logarithmic functions
use the methods of logarithmic differentiation and implicit differentiation to find derivatives

40
solve problems involving tangent and normal lines and the Mean Value Theorem
Note: The way to master calculus is to solve lots of calculus problems!
3.

Prescribed reading

The overview of chapter 2 of Smith on page 145 and sections 2.1, 2.2, 2.3, 2.4, 2.5, 2.6, 2.7, 2.8 and 2.9.
4.
4.1

The derivative
Introducing the derivative

The definition of the derivative can be approached in two different ways. One is geometrical (as a slope of a curve)
and the other is physical (as a rate of change). Both of these definitions can be used, but the emphasis should be on
using the derivative as a tool for solving calculus problems.
Remember, in high school, you used the first principle of differentiation to find the slope of a tangent line at a
particular point.
Definition: The slope m tan of the tangent line y = f (x) at x = a is:
m tan = lim

h0

f (a + h) f (a)
h

Example:
Find the slope of the tangent line to the curve y = x 2 + 1 at x = 1.
Solution:

8
m tan = lim

h0

= lim

h0

f (1 + h) f (1)
h
(1 + h)2 + 1 (1 + 1)
h

1 + 2h + h 2 + 1 2
h0
h

= lim

2h + h 2
h (2 + h)
= lim
h0
h0
h
h

= lim

_2

x
2

= lim (2 + h) = 2
h0

Thus at x = 1, m tan = 2.

41

MAT1512/1

You should also be familiar with the approach used in mechanics when dealing with problems involving average
and instantaneous velocity (rate of change) .
Average velocity :
v ave =
Instantaneous velocity :
v = lim

h0

total distance
total time

f (t + h) f (t)
h

Example:
Consider an object that is dropped from a height of 64 m. Its position is given as f (t) = 64 16t 2 . Find its average
velocity between
(a) t = 1 and t = 2
(b) t = 1.5 and t = 2
(c) t = 1.9 and t = 2
Also find its instantaneous velocity at t = 2.
Solution:

(a) v ave

64 16 (2)2 64 16(1)2
f (2) f (1)
=
=
= 48 m/s.
21
1

(b) v ave =
(c) v ave =

64 16 (2)2 64 16(1.5)2
f (2) f (1.5)
=
= 56 m/s.
2 1.5
0.5
64 16 (2)2 64 16(1.9)2
f (2) f (1.9)
=
= 48 m/s.
2 1.9
0.1

(d) The instantaneous velocity is given by:

f (2 + h) f (2)
(2 + h) 2
64 16 (2 + h)2 64 16 (2)2
lim
h0
h
64 16 4 + 4h + h 2 64 16 (2)2
lim
h0
h
2
64h 16h
16h (h + 4)
= lim
lim
h0
h0
h
h
lim 16 (h + 4) = 64 m/s.

v instantaneous = lim

h0

=
=
=
=

h0

42
4.2

Definition of the derivative

The derivative of the function f (x) (at a particular point where x = a) is defined as
f (a + h) f (a)
f (a) = lim
,
h0
h
provided that the limit exists. We say that f is differentiable at x = a when the above limit exists.
In general, the derivative of f (x) is the function f (x) given by
f (x + h) f (x)
f (x) = lim
,
h0
h
provided that this limit exists. Finding the first derivative by using the above method is called the first principle of
differentiation.
Note: If we write x = a + h, then h = x a and h will approach 0 if and only if x approaches a. Therefore another
way of defining the derivative is as follows:
f (x) f (a)
.
f (a) = lim
xa
x a
Let us try an example:
Use the definition of the derivative to find g (x) if g (x) = x 4 .

g (x + h) g (x)
(x + h)4 x 4
= lim
h0
h0
h
h
2
2
2
2
(x + h) x (x + h) + x
= lim
h0
h
h [2x + h] (x + h)2 + x 2
= lim
h0
h
= lim [2x + h] (x + h)2 + x 2

g (x) = lim

h0

= (2x) x 2 + x 2
= 4x 3 .

5.

Worked examples

In some instances, we do compute derivatives directly from the definition. However, such computations are tedious
and very time-consuming. Fortunately, several differential rules or basic differentiation formulas have been developed to find derivatives without using the definition directly. In order to introduce you to the basic differentiation
techniques, these rules are presented together with worked examples. You should go and read about these rules in
Smith and work through the examples, following each rule. You will find the rules on the following pages in Smith:
Derivative of a constant

p170

The power rule for positive integer powers of x


The constant multiple rule
The sum and difference rule
The power rule for negative integer powers of x

p 171
p 173
p 173
p 184

The product rule

p181

The quotient rule

p182

The chain rule

p189

43

MAT1512/1

Our collection of worked examples relating to chapter 2 of Smith divides naturally into eight sets, with some overlap,
namely:

I. Differentiation from first principles


II. Basic differentiation formulas (use of the power rule, product rule, quotient rule, chain rule, and combinations
of these rules to differentiate a variety of functions)
III. Derivatives of trigonometric functions and inverse trigonometric functions
IV. Derivatives of exponential and logarithmic functions
V. Logarithmic differentiation
VI. Implicit differentiation
VII. Tangent and normal lines
VIII. The Mean Value theorem

Attempt the problems appearing immediately after each set, once you have studied the relevant parts of Smith and
done some of the exercises there.

44
The table below (Table 3) shows how these worked examples and their solutions are organised.

Table 3

I.

Topic(s)

Sections in
Smith & Minton

Study guide
examples

I.

Differentiation from first


principles (or by using the
definition of the derivative)

Section 2.2 (examples


2.1 2.7)

15

II.

Basic differentiation
(use of the power rule,
higher-order
product rule,
quotient rule,
chain rule,
and combinations of these rules
to differentiate a variety of
functions)

Sections 2.2 2.5, 2.7, 2.8


Sections 3.1 3.5
Sections 3.6 3.7
Sections 4.1 4.2
Sections 4.3
Sections 5.1 5.4

631

III.

Derivatives of trigonometric
functions and
inverse trigonometric functions

Section 2.6 (examples


6.1 6.4 & 6.7)
Section 2.8 (examples
8.5 8.6)

3249

IV.

Derivatives of exponential and


logarithmic functions

Section 2.7 (examples


7.2 7.4

5363

V.

Logarithmic differentiation

Section 2.7 (example 7.6)

6474

VI.

Implicit differentiation
(second order)

Section 2.8: (examples


8.1 8.3 & 8.4)

7582

VII.

Tangent and normal lines

Sections 1.2 1.5

8392

VIII.

The Mean Value Theorem

Section 2.8 (example 9.3)

9395

Differentiation from first principles (definition of the derivative)


1. Find the derivative of 3x 2 + 2x 1 at x = 1.
2. If f (x) =
3. If f (x) =

1
x

(x = 0), find f (x) .

x for (x 0), find f (x) .

5052

45

MAT1512/1

Solutions:
1.
f (x) =
=
=
At x = 1, f (1) =

3 (x + h)2 + 2 (x + h) 1 3x 2 + 2x 1
f (x + h) f (x)
lim
= lim
h0
h0
h
h
2
2
2
3x + 6xh + h + 2x + 2h 1 3x 2x + 1
6xh + h 2 + 2h
= lim
lim
h0
h0
h
h
h (6x + h + 2)
lim
= lim 6x + h + 2 = 6x + 2.
h0
h0
h
6 (1) + 2 = 8.

2.
f (x + h) f (x)
h0
h
1
1

x (x + h) 1
x+h
x
= lim
= lim

h0
h0
h
x (x + h) h
h
1
1
1
= lim 2
= 2.
= lim 2
h0 x + xh h
h0 x + xh
x

f (x) = lim

3.

f (x + h) f (x)
x +h x
x +h x
x +h+ x
f (x) = lim
= lim
= lim

h0
h0
h0
h
h
h
x +h+ x
x +hx
h
1
= lim
= lim
= lim

h0 h
h0 h
h0
x +h+ x
x +h+ x
x +h+ x
1
.
=
2 x

4. Use the definition of the derivative to prove that g (0) = 0 if g (x) = x 2 |x| .
Solution:
By definition,
g (0) = lim

h0

g (0 + h) g (0)
,
h

(a) provided this limit exists. Now


g (0 + h) h (0)
h
2
h |h| 0
= lim
h0
h
= lim (h |h|)
lim

h0

h0

= 0.

5. Use the definition of the derivative to show that f (2) does not exist when f (x) = |x 2|.

46
(a)
lim+

h0

|(2 + h) 2| |2 2|
h0
h
|h|
= lim+
h0 h
h
(|h| = h since h > 0)
= lim+
h0 h
= 1,

f (2 + h) f (2)
h

lim+

(b) while similarly


lim

h0

f (2 + h) f (2)
h

|h|
h
h
= lim
h0 h
= 1.
=

lim

h0

(|h| = h since h < 0)

Since the left-hand and right-hand limits are not equal, it follows that
lim

h0

f (2 + h) f (2)
h

does not exist, that is, f is not differentiable at 2.


II.

Basic differentiation formulas

(a) The Power Rule


y = x n , then
d
(x n ) = nx n1
dx
dy
= nx n1
dx

If

i.e.
Examples:
Find the derivatives of the following:
6.

f (x) = x 8

d
3
x2
dx

7.

8.

Solutions:
7.

f (x) = 8x 81 = 8x 7

8.

m (x) = x 2 + 3x 2 m (x) =

d
d 2
2 2
2 1
2
3
x2 =
x 3 = x 3 1 = x 3 =
1
dx
dx
3
3
3x 3

7.
1
2x

1
2

3
3

2x 2

Examples involving absolute values:


Find the derivatives of the following:
9.

k (a) =

1
|a|

10.

k (s) = |s|

m (x) = x 2 + 3x 2

47

MAT1512/1

Solutions:
1
|a|
1
if
a

9. k(a) =

a>0

1 if a < 0.
a
Hence,

2 if a > 0

a
k (a) =

1
if a < 0.
a2

In fact, k (a) can be written as a single expression, namely k (a) =

1
.
a |a|

10.
k (s) = |s|

k (s) =

1 if s > 0
1 if s < 0.

Note: k (s) = |s| is not differentiable at s = 0.


(b) The Product Rule

If y = f.g, then y = f .g + f.g

or

d
f (x) g (x) = f (x) g (x) + f (x) g (x)
dx

Examples:
11.

Find f (x) if

f (x) = 2x 4 3x + 5

x2

x+

2
x

Solution:
f (x) =
=
12.

d
2
d
2
+ 2x 4 3x + 5
x2 x +
2x 4 3x + 5 x 2 x +
dx
x
dx
x

2
2
1
+ 2x 4 3x + 5 2x 2
8x 3 3 x 2 x +
x
x
2 x

Differentiate the following function:


y = (x + 1)10 (2x + 3)11 (4 x)12

48
Solution:

y = (x + 1)10 (2x + 3)11 (4 x)12


dy
d
d
= (x + 1)10 (2x + 3)11 (4 x)12 + (x + 1)10 (4 x)12 (2x + 3)11
dx
dx
dx
d
+ (2x + 3)11 (4 x)12 (x + 1)10
dx
d
d
= (x + 1)10 (2x + 3)11 12(4 x)11 (4 x) + (x + 1)10 (4 x)12 11(2x + 3)10
(2x + 3)
dx
dx
+ (2x + 3)11 (4 x)12 .10(x + 1)9
= 12(x + 1)10 (2x + 3)11 (4 x)11 + 22(x + 1)10 (2x + 3)10 (4 x)12
+ 10(x + 1)9 (2x + 3)11 (4 x)12 .

(c) The Quotient Rule:


If y =
or

13.

f
f .g f.g
=y
, then
g
g2

d
dx

Compute the derivative of

f (x) g (x) f (x) g (x)


f (x)
=
2
g (x)
g (x)

x2 2
.
x2 + 1

Solution:
d
dx

f (x) =
=

x2 2

x2 + 1 x2 2
x2 + 1

d
dx

x2 + 1

2x x 2 + 1 x 2 2 (2x)

x2 + 1

6x
x2 + 1

More examples:
Differentiate the following functions:

14.

2t + 1
g (t) =
2t + 1 + 1

Solutions:

15.

g (x) =

x +1
(x + 2)10

16.

y=

2x + 5
3x 2

49

MAT1512/1

14.
g(t) =

g (t) =

2t + 1

(2t + 1) 2
=

1
2t + 1 + 1
(2t + 1) 2 + 1
1
1
1
(2t + 1) 2 2 (2t + 1) 2 + 1
2
1
1
1
1
(2t + 1) 2 (2t + 1) 2 2 / (2t + 1) 2 + 1
2

1
=
.

2
2t + 1 2t + 1 + 1

15.
g (x) =

g (x) =

16.

x +1
(x + 2)10
1. (x + 2)10 (x + 1) (x + 2)9 .10.1
1 9x
=
.
20
(x + 2)
(x + 2)11

y =

dy
dx

(d) The Chain Rule

2x + 5
3x 2
19
(3x 2) 2 (2x + 5) 3
=
.
2
(3x 2)
(3x 2)2

If y = f g, then y = f (g) .g
or

d
f (g (x)) = f (g (x)) g (x)
dx

Examples:
17.

Differentiate y = x 3 + x + 1

19.

Find

5.

18.

Find

d
100 + 8t .
dt

dy
if y = (4 3x)9 .
dx

Solutions:
17.

18.

19.

Let u = x 3 + x + 1, then y = u 5 , and


dy
du
dy du
d
=
=
u5
dx
du dx
du
dx
d
4
x 3 + x + 1 = 5 x 3 + x + 1 3x 2 + 1
= 5u 4
dx

1
1
Let u = 100 + 8t, then
100 + 8t = (100 + 8t) 2 = u 2 , and
d
1
4
1 1 du
d
1
=
(u) 2 = u 2
(100 + 8t) =
du
2
dt
dt
2 100 + 8t
100 + 8t
y = (4 3x)9

dy
= 9 (4 3x)8 (3) .
dx

50
(e) Combinations of rules

In the following worked examples, combinations of rules are used to determine the required derivatives.
Find the first derivatives of the following. Do not simplify.

20.

f (x) = (5x + 11)20 (4 x)30

22.

f (x) =

1x
1 + x2

1
3
24. (y) = (2y 2 + 5y) 2
3
1

26. h( ) = ( 3 + 7)100 (2 3 + 7)200

21.

f (w) = (3w2 + 2) 3 (9w 1) 3

23.

f (x) =

25.

f (x) = (2x 3 + 3)10 (3x 2 + 2)20

1
x+
x

1
x
+
5 5x

27. y =

100

28. y = (1 6x) 3
Solutions:
20. f (x) = (5x + 11)20 (4 x)30

f (x) = 20(5x + 11)19 5(4 x)30 + (5x + 11)20 30(4 x)29 (1).
2

21. f (w) = (3w2 + 2) 3 (9w 1) 3


1

f (w) = 23 (3w2 + 2) 3 (6w)(9w 1) 3 + (3w2 + 2) 3 13 (9w 1) 3 9.

22. f (x) =

1x
=
1 + x2

f (x) =

23. f (x) =

1
2

1x
1 + x2

1
x+
x

f (x) = 100

1x
1 + x2

12

1
2

(1) 1 + x 2 (1 x) 2x
1 + x2

100

1
x+
x

99

1
1
3
2 x
2x 2

1
3
24. (y) = (2y 2 + 5y) 2
3
1 3
1
(y) = (2y 2 + 5y) 2 (4y + 5).
3 2

51

MAT1512/1

25. f (x) = (2x 3 + 3)10 (3x 2 + 2)20

f (x) = 10(2x 3 + 3)9 6x 2 (3x 2 + 2)20 + 20(3x 2 + 2)19 6x (2x 3 + 3)10 .


1

26. h() = ( 3 + 7)100 (2 3 + 7)200


1 2
2 4
1
1
1
1
h () = 100( 3 + 7)99 3 (2 3 + 7)200 + ( 3 + 7)100 200(2 3 + 7)199 ( 3 ).
3
3
27. y =

1
x
+
5 5x

dy
1
x
=5
+
dx
5 5x

1 1 2
x
5 5

28. y = (1 6x) 3
dy
2
1

(6).
=
dx
3 (1 6x) 13
Also do the following exercises:

29. Show that y = 2 |x| if y = x |x| .


30. Let g (x) , h (x) and k (x) be differentiable functions such that
f (x) = g (x) h (x) k (x)
for all x. Use the product rule for the derivative of the product of two functions to write down the derivative
of f in terms of g (x) , h (x) and k (x) and their derivatives.
31. Use the chain rule and product rule to derive the quotient rule for derivatives.

Solutions:
29. Case I

Suppose x > 0. Then |x| = x, so

y = x2

and
y = 2x = 2 |x| .
Case II

Suppose x < 0. Then |x| = x, so


y = x 2

and
y = 2x = 2 (x) = 2 |x| .

52
Case III

Suppose x = 0. If f (x) = x|x|, then

f (0 + h) f (0)
h

lim

h0

h |h| 0
h
= lim |h|
= lim

h0
h0

= 0 = 2 |0| .

Hence, for all values of x we have that


y = 2 |x| .
30. Let
f (x) = (x)k(x),
where
(x) = g(x)h(x).
Then
f (x) =

(x)k(x) + (x)k (x)

= (g (x)h(x) + g(x)h (x))k(x) + (x)k (x)

= g (x)h(x)k(x) + g(x)h (x)k(x) + g(x)h(x)k (x).


31. Let f (x) =

g (x)
, where g and h are differentiable functions. Now
h (x)
f (x) = g(x) (h(x))1

so
1
h (x)
(h (x))2
(by the product and chain rules)
g (x) g (x) h (x)

=
h (x)
(h (x))2
h (x) g (x) g (x) h (x)
=
.
(h (x))2

f (x) = g (x) (h(x))1 + g(x)

III.

Derivatives of trigonometric functions and inverse trigonometric functions

(a) Derivatives of trigonometric functions


Below are limits which will help you find some of the trigonometric derivatives.

lim cos = 1,

1 cos
cos 1
= lim
=0
0
0

lim sin = 0,

sin

= lim
=1
0
0 sin

lim

lim

53

MAT1512/1

A very important table


You should know these derivative formulae by heart!
Note: These derivative formulae can also be derived from first principles.
d
(sin x) = cos x
dx

d
(cot x) = csc2 x
dx

d
(cos x) = sin x
dx

d
(sec x) = sec x tan x
dx

d
(tan x) = sec2 x
dx

d
(csc x) = csc x cot x
dx

Here is an example worked out from first principles:


Prove that

d
sin x = cos x.
dx

Solution:
If f (x) = sin x, then
f (x + h) f (x)
f (x) = lim
h0
h
sin (x + h) sin (x)
= lim
h0
h
sin x cos h + cos x sin h sin x
= lim
h0
h
cos h 1
sin h
+ cos x
= lim sin x
h0
h
h
cos h 1
sin h
= lim sin x. lim
+ lim cos x. lim
h0
h0
h0
h0
h
h
= sin x.0 + cos x.1
= cos x

Now determine the derivatives of the following functions:


32. f (x) = x 5 cos x

33.

f (x) = sin

2x
x +1

Solutions:
32.
d
d
x 5 cos x =
x5
dx
dx
= 5x 4 cos x x 5 sin x.

f (x) =

cos x + x 5

d
(cos x)
dx

{product rule}

54

33. The derivative of f (x) = sin

2x
x +1

is

2x
x +1
2x
= cos
x +1
2x
= cos
x +1

f (x) = cos

d
2x
dx x + 1
2 (x + 1) 2x (1)
(x + 1)2
2
.
(x + 1)2

{chain rule}
{quotient rule}

Now go through the rest of the worked examples. Find the first derivative. Do not simplify.
34. h(t) = sec t + tan t
36. g ( ) =
38.

sin

35. y = x csc x
37. h(t) =

3t + cos t
f (t) =
t +3

39. y =

sin t
tan t + 1

cos x
1 + sin x

41. k (x) = cot2 (3x)


40. h (w) = sin3 (cot w)

42. g(x) = cos cx + cos c,where is a constant


3

43. g() = sec 2 (1 )


1

44. g(x) = tan4 (tan x)


2x + 5
3x 2

45. h(t) = (t 3 + 1). tan t

46. y =

47. y = cos(3 x/2)

48. y = cos3 x
5

49. y = [sin(x + 5)] 4


Solutions:
34. h(t) = sec t + tan t
h (t) = (sec t)(tan t) + sec2 t.
35. y = x cosc x
y = x.( cosc x cot x) + (cosc x).1.
sin

cos (sin ) (1)


.
g () =
2

36. g() =

55

MAT1512/1

sin t
tan t + 1
cos t (tan t + 1) sin t sec2 t
h (t) =
.
(tan t + 1)2

37. h(t) =

3t + cos t
38. f (t) =
t +3

1
2 3t+cos t

f (t) =

(3 sin t) (t + 3)
(t + 3)2

3t + cos t 1

cos x
1 + sin x
(1 + sin x) ( sin x) cos x (0 + cos x)
y =
.
(1 + sin x)2

39. y =

40. h(w) = sin3 (cot w)

h (w) = 3 sin2 (cot w). cos(cot w).(cosec2 w).

41. k(x) = cot2 (3x)

k (x) = 2 cot(3x).(cosec2 (3x)).3.

cos cx + cos c,where is a constant


1
g (x) =
( cos cx. cot cx).
2 cos cx + cos c

42. g(x) =

43. g() = sec 2 (1 )


3
1
g () = sec 2 (1 ) sec(1 ) tan(1 ) (1).
2
44. g(x) = tan4 (tan x)

g (x) = 4 tan3 (tan x). sec2 (tan x). sec2 x.


1

45. h(t) = (t 3 + 1) tan t


1 4
1
h (t) = t 3 tan t + (t 3 + 1) sec2 t.
3
46. g() = cos2 ( 2 )

g () = 2 cos( 2 ) ( sin( 2 )) 2.

47. y = cos(3 x/2)


dy
d

= sin(3 x/2) (3 x/2)


dx
dx
= (3/2) sin(3 x/2).

56
48. y = cos3 x
dy

= 3(cos2 x)( sin x).


dx
5

49. y = [sin(x + 5)] 4


1
dy
5

= [sin(x + 5)] 4 [cos(x + 5)].


dx
4
(b) Derivatives of inverse trigonometric functions
These are useful in applications and are essential for solving problems. The rules or definitions are as follows:
1
d
, for 1 < x < 1
sin1 x =
dx
1 x2
1
d
cos1 x =
, for 1 < x < 1
dx
1 x2
1
d
tan1 x =
dx
1 + x2
1
d
cot1 x =
dx
1 + x2
1
d
sec1 x =
, for |x| > 1

dx
|x| x 2 1
1
d

csc1 x =
, for |x| > 1
dx
|x| x 2 1
Examples:
Compute the derivatives of the following functions:
50. cos1 3x 2

51.

sec1 x

52.

Solutions:
50.

d
cos1 3x 2 =
dx

51.

d
sec1 x
dx

52.

6x
d
.
3x 2 =
2 dx
1 9x 4
1 3x 2
1

= 2 sec1 x

d
1
tan1 x 3 =
dx
1 + x3

d
1
.
sec1 x = 2 sec1 x

dx
|x| x 2 1

3x 2
d 3
=
.
x
2 dx
1 + x6

tan1 x 3

57
IV.

MAT1512/1

Derivatives of exponential and logarithmic functions

(a) The exponential function

Definition:

The inverse function of the function ln x is known as the expo


nential function and is indicated by exp x (or e x ).
Properties of the exponential function
(a) The graph of e x is the mirror image of the graph of ln x in the line y = x. Therefore the graph looks like this:
y
y =e

y =x

y = ln x

x
0

(b) exp x is defined for all x R, in other words


Dom (exp x) = R.
(c) Im (exp x) = {y|y > 0}
(d)

d x
(e ) = e x
dx

(e) e x+y = e x e y
(f) e xy = e x /e y
(g) er x = (e x )r
(h) If x , then e x .

58
(i) If x , then e x 0.

(b) The logarithmic function


Definition:

The natural logarithmic function y = ln x is the inverse


function of the exponential function.
Properties of the ln function
(a) The graph of ln x looks like this:
y
y =x

y = ln x

x
0

(b) ln x is defined for all x > 0, in other words


Dom (ln x) = {x| x > 0} .
(c) Im (ln x) = R
(d) ln x < 0 if 0 < x < 1,
ln 1 = 0,
ln x > 0 if x > 1.
(e)

1
d
(ln x) =
dx
x

(f) ln(x y) = ln x + ln y, if x, y > 0


(g) ln

x
= ln x ln y, if x, y > 0
y

59

MAT1512/1

(h) ln x r = r ln x, if x > 0, r rational


(i) logb a =

logc a
logc b

(j) If x , then ln x .
(k) If x 0+ , then ln x .
Remark:
The following mistake is often made when applying rule (h):
ln x r = r ln x

(this is correct)

but
(ln x)r = r ln x

(incorrect).

The index r must be an index of x, but not of ln x.


The inverse of the exponential function e x is called the natural logarithmic function y = ln x. In this section, we
study the natural logarithm both as a differentiable function and as a device for simplifying calculations.
(c) Examples of the exponential function
Find the derivatives of the following:
53. f (x) = e

54. y = e x cos x
55. y = tan e3x2
56. y =

e3x
1 + ex

57. y = e x+e

Solutions:
53. f (x) = e

f (x) = e

12 x

12

e x
= .
2 x

54. y = e x cos x
y = e x cos x (1. cos x + x ( sin x)) = e x cos x (cos x x sin x) .

60
55. y = tan e3x2
y = sec2 e3x2
56. y =
y =

e3x2 .3 = 3e3x2 sec2 e3x2 .

e3x
1 + ex
(1 + e x ) e3x .3 e3x (0 + e x (1))
e3x
3e3x + 3e4x e4x
3e3x + 2e4x
=
=
=
.
1 + ex
(1 + e x )2
(1 + e x )2
(1 + e x )2

57. y = e x+e

y = e x+e (1 + e x (1)) = e x+e (1 + e x ) .


(d) Examples of the logarithmic function
Find the derivatives of the following:
58. f (x) = x 2 ln 1 x 2
59. f (x) = log3 x 2 4
60. f (x) = log10

x
x 1

61. y = ln x 1 x 2 sin x
62. y = x sin x
63. y = (sin x)x
Solutions:
58. f (x) = x 2 ln 1 x 2
f (x) = 2x ln 1 x 2 + x 2

2x 3
1
2

2x)
=
2x
ln
1

.
(0
1 x2
1 x2

59. f (x) = log3 x 2 4

ln x
,
ln a
ln x 2 4
2
so f (x) = log3 x 4 =
ln 3
1
2x
1
1
2x
= 2
f (x) =
.
(2x 0) =
2
2
ln 3 x 4
ln 3 x 4
x 4 ln 3
Note that (or recall that) loga x =

61
x
x 1
ln x
Recall: loga x =
,
ln a

60. f (x) = log10

x
1
1
x
x
x 1
=
=
ln
=
[ln x ln (x 1)]
so f (x) = log10
x 1
ln 10
ln 10
x 1
ln 10
1
1 (x 1) 1.x
1
x 1x
1
f (x) =
=
=
ln 10
x (x 1)
ln 10 x (x 1)
x (x 1) ln 10
ln

61. y = ln x 1 x 2 sin x

1
ln 1 x 2 + ln sin x
2
1
1 1
1
so y = +
(cos x (1))
(0 2x) +
2
x
2 1x
sin x
cos x
1 1 2x
that is y = +
+
x
2 1 x2
sin x
1
x
+ cot x.
= +
x
1 x2
y = ln x +

62. y = x sin x
You should take natural logs on both sides first and get:
ln y = ln x sin x
ln y = sin x ln x
Now take the derivatives of both sides:
1
1
y = cos x. ln x + sin x.
y
x
sin x
sin x
= x sin x cos x. ln x +
.
y = y cos x ln x +
x
x
63. y = (sin x)x
Take logs on both sides and get:
ln y = x ln (sin x)
1
1
and y = 1. ln (sin x) + x
. cos x ,
y
sin x
cos x
so y = y ln (sin x) + x.
sin x
= (sin x)x [ln (sin x) + x. cot x] .

MAT1512/1

62
V.

Logarithmic differentiation

(a) The simplification of functions


As a result of its properties, a function involving the logarithmic function can be simplified before it is differentiated,
as becomes clear in the following example:
Example:
Determine f (x) if f (x) = ln

x1
x2

Solution:
Now
f (x) = ln
=
=

x 1
x2

1/3

(using properties (f), (g) and (h) of logarithmic functions described earlier on)

1
ln (x 1) ln x 2
3
1
2
ln(x 1) ln x.
3
3

Therefore
f (x) =
=

2
1

3
(x 1)
x + 2
.
3x (x 1)
1
3

1
x

Even if the logarithmic function itself does not occur in the expression, it can be used for the simplification of
functions. We apply the logarithmic function to both sides of the equation and then differentiate (see below).
Remark:
Always remember that when applying the chain rule to

d
dx

(ln f (x)) , it becomes

1
d
f (x) .
(ln f (x)) =
dx
f (x)
(the derivative of the first function multiplied by the derivative of the second function).
For example:

=
Examples:
64. Determine y if y = x 2 e2x cos x 3 .

d
(ln sin x)
dx
1
cos x.
sin x

63

MAT1512/1

2
x (1x 2 )
65. Determine f (x) if f (x) =
.
2

1+x

66. Determine f (x) if f (x) = log3

x 2 +4
3

(x1) 2

Solutions:
64. We have that
y = x 2 e2x cos x 3
By applying the logarithmic function on both sides, we find that
ln y = ln x 2 e2x cos x 3

= 2 ln x + 2x + ln cos x 3 .

By differentiating we obtain
2
1
1
sin x 3 3x 2 .
y = +2+
y
x
cos x 3
Therefore
y

2
+ 2 3x 2 tan x 3
x
2
+ 2 3x 2 tan x 3 .
= x 2 e2x cos x 3
x
= y

65.
x 1 x2
f (x) =
1 + x2

Once again we apply the logarithmic function on both sides and thus obtain
x 1 x2
ln f (x) = ln
1 + x2

= ln x + 2 ln(1 x 2 )

1
ln(1 + x 2 ).
2

By differentiation we therefore obtain that


1
x
4x
1

.
f (x) = +
2
f (x)
x
1x
1 + x2

64
Therefore
f (x) =

x 1 x2

1 + x2

=
=
=
=

x
4x
1

2
x
1x
1 + x2

4x 2 1 x 2
x2 1 x2
1 x2

3
1 + x2
1 + x2
1 + x2 2
1 x2

1 x2

1 + x 2 4x 2 1 x 2
1 + x2

1 x2

1 5x 2 4x 4
1 + x2

3
2

3
2

1 + x2 x2 1 x2

1 + x 2 4x 2 1 + x 2 x 2 1 x 2
1 x2

1 + x2

3
2

66.
f (x) = log3
=
Now using the fact that
loga x =

x2 + 4
3

(x 1) 2

1
3
log3 (x 2 + 4) log3 (x 1)
2
2

logb x
[Formula 5.4 on p 55 in Smith]
logb a

we find that
f (x) =
=

3 loge (x 1)
1 loge x 2 + 4

2
loge 3
2 loge 3
2
1 ln x + 4
3 ln (x 1)

.
2
ln 3
2 ln 3

If we keep in mind that ln 3 is a constant, it follows that


f (x) =
=

1
1
3
1
2
2x

2 ln 3 x + 4
2 ln 3 x 1
3
x

.
x 2 + 4 ln 3 2 (x 1) ln 3

(b) Functions of the form f (x) = g(x)h(x)


We employ logarithmic differentiation to differentiate functions of the form
f (x) = g(x)h(x)
Just as in (a), we do this by applying the logarithmic function on both sides and then making use of the properties
of the logarithmic function.
Then
ln f (x) = ln g(x)h(x) .

65

MAT1512/1

Therefore
ln f (x) = h(x). ln g(x).
Example:
Determine f (x) if f (x) =

Solution:
By applying the logarithmic function on both sides, we obtain
x
ln f (x) = ln x

=
x ln x (See the remark earlier on under (a) The simplification of functions.)
By differentiating, we obtain from the chain rule and the product rule that

1 1
1 1 1
1
x x 2 + ln x x 2
f (x) =
f (x)
2
x 2

1
1
=
+ ln x.
2 x
2 x
Therefore
f (x) =
=
=

f (x)

1
1
+ ln x
2 x
2 x

1
1 + ln x
2 x

x
1 + ln x .
2 x

Now attempt the following yourselves. (The answers are given at the end, so that you can make sure that you
understand this important section well.)
Determine f (x) below:
67. f (x) = ln x +

68. f (x) = (ln x)

69. f (x) = ln

70. f (x) = x ln

x 2 + 1 (be careful with this exercise!)

x2 1
x2 + 1
2

+ 7x 3

66
1

71. f (x) =

x (x 1) 3
2

(x + 1) 3

72. f (x) = a 3x
73. f (x) = x e

x 2

74. f (x) = log10 (log10 x)


Solutions:
67.

1
x2

(Use the chain rule on the right-hand side and simplify.)

+1

(ln x)
68.

ln ln x
1
+
ln x
2

69.

4x
3 x4 1

70.

2 ln x
x ln
x

71.

2x 2 + 3x 3
2

2 ln 7
1

3x 3

7x 3

3 (x 1) 3 (x + 1) 3
2

72. 6x a 3x ln a
73. x e

74.

VI.

x 2

ex

1
x

2x ln x

1
x ln x ln 10
Implicit differentiation

This technique is used when the function contains variables that cannot be separated from each other. This process
of differentiating both sides of the equation with respect to x and then solving for y (x) is called implicit differentiation.
Example:
Find y (x) for x 2 + y 3 2y = 3. Then find the slope of the tangent line at the point (2, 1).

67

MAT1512/1

Solution:
Differentiate both sides

d
d
x 2 + y 3 2y =
(3)
dx
dx

with respect to x :

2x + 3y 2 y (x) 2y (x) = 0
3y 2 y (x) 2y (x) = 2x

Solve for y (x) :

y (x) =

2x
3y 2 2

Find the slope of the


tangent line at (2, 1) :

y (2) =

4
= 4
32

Therefore the equation of the tangent line is


y 1 = 4 (x 2)
Now try the following exercises:

75. Find y (x) for x 2 y 2 2x = 4 4y. Then find the slope of the tangent line at the point (2, 2) .
76. Determine the derivative of tan1 x.
77. Van der Waals equation for a specific gas is

P+

5
(V 0.03) = 9.7.
V2

dV
Using the volume V as a function of pressure, use implicit differentiation to find the derivative
at the
dP
point (2, 2).
78. Find the equations of the tangent and the normal lines to the curve of
2x y + sin y = 2 at the point 1,
79. Find

dy
if tan x y 2 + 3y = 2x y.
dx

80. Find

dy
if y 2 x 2 + y + cos (x y) = 0.
dx

81. Find

dy
if sin (x + y) = y 2 cos x.
dx

82. Find the equations for the tangent and the normal lines to the curve of
x sin (2y) = y cos (2x)
at the point


,
4 2

.
2

68
Solutions:

75.

d
x 2 y 2 2x
dx
dy
2
2x y 2 + x 2 2y
dx
dy
2x 2 y + 4
dx
dy
dx
dy
= mT
d x (2,2)

d
(4 4y)
dx
dy
04
dx

2 2x y 2

=
=

2 2x y 2
2x 2 y + 4
7
2 2 (2) (2)2
14
=
=
2
12
6
2 (2) (2) + 4

The equation of the tangent line is

y (2) =
that is

y+2 =

that is

y =
=
=
=

7
(x 2)
6
14
7
x
6
6
14
7
x
2
6
6
7
14 12
x

6
6
6
26
7
x
6
6
13
7
x .
6
3

76. The derivative of tan1 x:


Note: Since tan x is differentiable, tan1 x is also differentiable. To find its derivative, let y = tan1 x. Then
tan y = tan tan1 x tan y = x.
Differentiate this equation implicitly with respect to x to get:
sec2 y

dy
= 1.
dx

Then

dy
1
1
1
.
=
=
=
dx
sec2 y
1 + tan2 x
1 + x2

Therefore

d
1
.
tan1 x =
dx
1 + x2

69

MAT1512/1

77.
=

9.7

0.15
V2

9.7

d
P V 0.03P + 5V 1 0.15V 2
dP

dV
dV
+ 2 (0.15) V 3
dP
dP

dV
dP

0.03 V

dV
dP

0.03 V
5
0.3
P 2+ 3
V
V

(2,2)

0.03 (2)
5
0.3
2
+
2
(2)
(2)3

0.03 + 2
5 0.3
2
4
8

P V 0.03P +

5
V2

(V 0.03)

P+

1V + P

dV
dP

0.03 + 5 (1) V 2

5
V

0.3
5
+ 3
2
V
V

dV
dP

=
78. 2x y + sin y = 2

Differentiate both sides:


dy
2y + 2x dx
+ cos y

dy
dx

(2x + cos y)

dy
dx

2y

dy
dx

2y
.
2x + cos y

At (1, 2 ):

2 2
dy

=
= .
dx
2 + cos 2
2

Equation of the tangent line:


y

= 2 (x 1) y = 2 x + .

Equation of the normal line:


y

= 2 (x 1) y =

2
x

+ 2 .

203/100
812
=
.
57/80
150

70
79. tan x y 2 + 3y = 2x y

Differentiate both sides:


dy
dy
]+3
dx
dx
dy
2x y sec2 (x y 2 ) 2x + 3
dx
dy
dx

sec2 (x y 2 )[y 2 + 2x y

dy
2y + 2x dx

2y y 2 sec2 (x y 2 )

2y y 2 sec2 (x y 2 )
.
2x y sec2 (x y 2 ) 2x + 3

80. y 2 x 2 + y + cos (x y) = 0
Differentiate both sides:
2y

dy 2
dy
dy
x + 2x y 2 +
sin(x y) y + x
dx
dx
dx
dy
2yx 2 + 1 x sin(x y)
dx
dy
dx

2x y 2 + y sin(x y)

y sin(x y) 2x y 2
.
2x 2 y + 1 x sin(x y)

81. sin (x + y) = y 2 cos x

Differentiate on both sides:


dy
dx
dy
cos(x + y) + cos(x + y)
dx
dy
cos(x + y) 2y cos x
dx
dy
dx
cos(x + y) 1 +

dy
cos x + y 2 ( sin x)
2y dx

dy
cos x y 2 sin x
2y dx

y 2 sin x cos(x + y)

y 2 sin x cos(x + y)
.
cos(x + y) 2y cos x

82. x sin (2y) = y cos (2x)

Differentiate both sides:


sin 2y + x cos 2y

dy
dx

dy
2x cos 2y cos 2x
dx
dy
dx

dy
cos 2x 2y sin 2x
dx

2y sin 2x sin 2y

2y sin 2x + sin 2y
.
cos 2x 2x cos 2y

At the point ( 4 , 2 ) the slope is:


sin( 2 ) + sin

dy
=
= 2 = 2.

2
dx
cos( 2 ) 4 cos
4
Therefore the equation of the tangent at the point ( 4 , 2 ) is:

= 2(x )
2
4
y = 2x,

71
and the equation of the normal line is:

VII.

MAT1512/1

1
5
y= x+
.
2
8

Tangents and normal lines

From your high-school mathematics, you should know that differentiating the equation of a curve gives you a
formula for the gradient (m) of the curve. The gradient of a curve at a point is equal to the gradient of the tangent at
that point. For example, in order to find the equation of the tangent to the curve y = x 3 at the point (2, 8), you first
have to determine the derivative: m = y = 3x 2 .
The gradient of the tangent at the point (2, 8) when x = 2 is 3 (2)2 = 12.
8y
y
The equation for the tangent line is: y = 12x 16 since
=
= 12 .
x
2x
The normal to the curve is the line which is perpendicular (at right angles) to the tangent to the curve at that point.
If two lines are perpendicular, then the product of their gradients is 1. So if the gradient of the tangent at the point
1
1
(2, 8) of the curve y = x 3 is 12, then the gradient of the normal is , since 12 = 1. Now try to work
12
12
through the following problems involving tangents and normal lines.
Examples:
83. Two curves are said to be orthogonal at a point where the curves intersect if their tangents are perpendicular to
each other at the point of intersection. Show that the curves y = sin(2x) and y = sin(x/2) are orthogonal
at the origin.
84. Find the x- and y-intercepts of the line that is tangent to the curve y = x 3 at the point (2, 8).
85. Does the graph of the function
y = 2x + sin x
have any horizontal tangents in the interval 0 x 2? If so, where? If not, why not?
86. Find equations for the lines that are tangent and normal to the curve f (x) = 1 + cos x at the point (/2, 1).
87.

(a) Find the equation for the tangent to the graph of


h(x) =

4
(1 + x)2

which makes an angle of 45 with the x-axis.


[NB Assume that the scales along the x- and y-axes are the same. Angles are measured counterclockwise
from the positive x-axis.]
(b) What is the equation of the normal to the curve of h at the point of tangency for the tangent considered
in (a) above?
88. Find the points on the curve y = 2x 3 3x 2 12x + 20 where the tangent is parallel to the x-axis. What are
the equations for these tangents?

72
89. Determine the equation for the normal to the curve of
f (x) =

x2

1
+1

at the point x = 1.
90. Determine the equations for the tangents to the curve of y = x 3 6x + 2 which are parallel to the line
y = 6x 2.
91. The line normal to the curve y = x 2 + 2x 3 at (1, 0) intersects the curve at what other point?
92. What are the equations for the tangents to the curve f (x) = x 2 + 5x + 9 which pass through the origin?
Solutions:
83.
y = sin(2x)
d
dy
= (cos(2x)) (2x)

dx
dx
= 2 cos(2x),
so

dy
dx

x=0

= 2 1 = 2.

Also,
x
y = sin( )
2
x d x
dy
= cos( )

dx
2 dx x
1
x
= cos( ), 2
2
2
so

Since

dy
dx

x=0

1
1
= 1= .
2
2

1
(2) = 1, the given curves are orthogonal at the origin.
2

84. Let
f (x) = x 3 .
Then
f (x) = 3x 2 ,
so the gradient at the point (2, 8) is f (2) , that is 12. The equation of the tangent at the point (2, 8)
is therefore given by
y (8) = 12(x (2))

73

MAT1512/1

or
y = 12x + 16

= 4(3x + 4).

4
The x-intercept of this tangent is and its y-intercept is 16.
3
85. Let
f (x) = 2x + sin x.
Then
f (x) = 2 + cos x.
Now the gradient of a horizontal line is 0, so if the graph of f is to have a horizontal tangent at the point
(x, f (x)), then the value of f (x) must be 0. We must therefore solve the equation
f (x) = 0
for all values of x occurring in the interval 0 x 2. But if
2 + cos x = 0
then
cos x = 2,
which is impossible. Hence the graph of f has no horizontal tangents.
86.
f (x) = 1 + cos x

f (x) = sin x.

, 1 is f
Hence the gradient at the point
2
is therefore

, that is 1. The equation of the tangent at the point


,1
2
2
y 1 = 1 x

or
y = x +
The gradient of the normal at the point
therefore

+ 1.
2

, 1 is 1. The equation for the normal at the point


2
y1=1 x

or
y=x

+ 1.
2

, 1 is
2

74
87.

(a) The tangent which makes an angle of 45 with the x-axis has a gradient of 1. To find the x-coordinate
of the corresponding point of tangency we let
h (x) = 1
and solve for x. Now
h(x) = 4(1 + x)2

h (x) = 8(1 + x)3 ,


and

8
= 1
(1 + x)3
x = 3.

We have that
h(3) =

4
= 1.
(1 3)2

The tangent thus passes through the point (3, 1) and has a gradient of 1. Its equation is
y = 1.(x + 3) + 1
that is
y = x + 4.
(b) We obtain the equation for the required normal:
y = 1.(x + 3) + 1
that is
y = x 2.
88.
y = 2x 3 3x 2 12x + 20

= 6x 2 6x 12.

Now
6x 2 6x 12 = 0

6(x 2 x 2) = 0

6(x 2)(x + 1) = 0

x = 2 or x = 1.

If x = 2 then y = 2 8 3 4 24 + 20 = 0.

If x = 1 then y = 2 3 + 12 + 20 = 27.

The required points are (2, 0) and (1, 27). The equations for the tangents are y = 0 and y = 27 respectively.

75

MAT1512/1

89.
1
= x2 + 1
+1
f (x) = (x 2 + 1)2 2x.
1
f (1) = ,
2
f (x) =

x2

so the gradient of the normal at x = 1 is 2. Now


1
f (1) = ,
2
so the equation for the normal at x = 1 is
y

1
= 2(x 1).
2

90. The slope of the tangent to the curve of y = x 3 6x + 2 is given by y =

dy
. Now
dx

y = 3x 2 6.
Since the tangent must be parallel to y = 6x 2, we must have that y is equal to the slope of y = 6x 2, that
is
y = 6
2

3x 6 = 6
3x 2 12 = 0

x2 4 = 0

x = 2.
The corresponding y-values are
y = (2)3 6(2) + 2
= 8 + 12 + 2 = 6

for x = 2, and
y = (2)3 6(2) + 2

= 8 12 + 2 = 2

for x = 2.
Therefore the tangent points are (2, 6) and (2, 2).
The equation for a line with slope m which passes through (xo , yo ) is given by
y yo = m(x xo ).

76
The tangent through (2, 6) therefore has the equation
y 6 = 6(x (2))
= 6x + 12

that is
y = 6x + 18.
The tangent through (2, 2) has the equation
y (2) = 6(x 2)
that is
y + 2 = 6x 12
that is
y = 6x 14.
91. The normal to the curve y = x 2 + 2x 3 at the point (1, 0) is perpendicular to the tangent at that point.
Hence, if we know the gradient of the tangent we can calculate the gradient of the normal. Now if
f (x) = x 2 + 2x 3
then
f (x) = 2x + 2.
Thus
f (1) = 4,
1
which is the gradient of the tangent. Hence the gradient of the normal is . Since the normal passes through
4
the point (1, 0), its equation is
1
y 0 = (x 1)
4
that is

1
1
y= x+ .
4
4

1
1
To find the points of intersection of the curve y = x 2 + 2x 3 and the line y = x + , let
4
4
1
1
x 2 + 2x 3 = x + .
4
4
Then
4x 2 + 8x 12 = x + 1
so
4x 2 + 9x 13 = 0,

77

MAT1512/1

that is
(x 1)(4x + 13) = 0.
Hence x = 1 or x =

13
13
. If x = then
4
4
1
1
13 1
17
x+ =
+ = .
4
4
16 4
16

The other point of intersection of the normal and the curve is therefore

13 17
.
,
4 16

92. The equation of the tangent at a point (, f ()) is


y f () = f ()(x ),
that is
y = f () x f () + f ().
The tangent will pass through the origin only if the yintercept is 0, that is
f () f () = 0,
and then the equation of the tangent becomes
y = f () x.

...()

Hence the solutions of the equation


f (x) x f (x) = 0
will give the x-coordinates of the points of contact between the tangents and the curve, and then the equations of the corresponding tangents can be determined from ().
Now
f (x) = x 2 + 5x + 9

f (x) = 2x + 5;
and

f (x) x f (x) = 0

x 2 + 5x + 9 x(2x + 5) = 9 x 2 = 0
x = 3 or x = 3.

We have that f (3) = 11 and f (3) = 1. Hence the required tangents have the formulae y = 11x and
y = x.

78
VIII.

The Mean Value Theorem

In this section we discuss the Mean Value Theorem. Please read carefully through sections 2.9 and 4.4 of chapter
2 in Smith. We would advise you to work through the examples in order to become familiar with how the Mean
Value Theorem is applied. In section 2.9 you have to understand theorem 9.1 and theorem 9.4. You have to be able
to apply these theorems. Please work through examples 9.1 to 9.3 in this section.
The Mean Value Theorem is one of the most important tools in calculus. It states that if f (x) is defined and
continuous on the interval [a, b] and differentiable on (a, b), then there is at least one number c in the interval (a, b)
(that is a < c < b) such that
f (b) f (a)
.
f (c) =
ba

The special case when f (a) = f (b) is known as Rolles Theorem. In that case, we have f (c) = 0. In other words,
there exists a point in the interval (a, b) which has a horizontal tangent. In fact, the Mean Value Theorem can be
stated in terms of slopes.
The number

f (b) f (a)
ba
is the slope of the line passing through (a, f (a)) and (b, f (b)), (see Figure 4).
Figure 4

b
a

Example:
Let f (x) =

f (b) f (a)
1
1
1
, a = 1 and b = 1. We have
=
x
ba
1
1

2
= 1.
2

1
On the other hand, for any c [1, 1] , not equal to 0, we have f (c) = 2 = 1.
c
f (b) f (a)
does not have a solution for c. This does not contradict the Mean Value
So the equation f (c) =
ba
Theorem since f (x) is not continuous on [1, 1].

79

MAT1512/1

Note: The derivative of a constant function is 0. You may wonder whether a function with a derivative of zero is
constant the answer is yes. To show this, we let f (x) be a differentiable function on an interval I, with f (x) = 0,
f (b) f (a)
= f (c) for some
for every x I . Then for a and b in I , the Mean Value Theorem implies that
ba
point c between a and b. So our assumption implies that f (b) f (a) = 0. (b a). Thus f (b) = f (a) for any a
and b in I, which means that f (x) is a constant function.
Worked Examples:
For the following functions, find a value of c which complies with the Mean Value Theorem.
93. f (x) = x 2 + 1, on the interval [2, 2] .
94. f (x) = x 3 + x 2 , on the interval [0, 1] .
95. f (x) = sin x, on the interval 0, 2 .
Solutions:
93. Since f (x) = x 2 +1 is a polynomial, f (x) is continuous on [2, 2] and differentiable on (2, 2). According
to the Mean Value Theorem, this means that there is a number c in (2, 2) for which
f (c) =

22 + 1 (2)2 + 1
f (2) f (2)
55
0
=
=
= = 0.
2 (2)
4
4
4

To find this number c, we obtain f (c) and then set f (c) = 0.


f (c) = 2c and 2c = 0, so c = 0

Notice that c = 0 [2, 2]

c = 0.

94. f (x) = x 3 + x 2 is a polynomial, continuous on [0, 1] and differentiable on (0, 1). According to the Mean
Value Theorem, there is a number c in (0, 1) for which
f (c) =

f (1) f (0)
20
= 2.
=
1 (0)
1

But f (c) = 3c2 + 2c.

So, f (c) = 3c2 + 2c = 2 3c2 + 2c 2 = 0.


Using the quadratic formula we get

2 28
2 2 7
22 4 (3) (2)
=
=
c =
2 (3)
6
6

1 7
1 7
1 + 7
c =
c=
or c =
3
3
3
c 1.22 or c 0.55.
2

But since c 1.22


/ [0, 1] , we only accept the alternative c 0.55 that is

1 + 7
c=
.
3

80
95. f (x) = sin x, on the interval 0, 2 .
As f (x) = sin x is a trigonometric function, it means that f (x) is continuous on 0, 2 and differentiable on
0, 2 . The conditions for the Mean Value Theorem hold, and so there exists c 0, 2 such that
f (c) =

f (0)
sin
=
0

2 sin (0)

10

.
2
But f (x) = cos x, f (c) = cos c and c has to be in the first quadrant. Thus
2
c = cos1
0.88,

,
so c = 0.88 0,
2
2
that is c = cos1
0.88.

So c 0.88 0,

2
.

81

MAT1512/1

Key points
You should by now have an understanding of the notion of differentiability and should also be able to compute the
derivatives of different functions using basic differential formulas such as the power, product, quotient and chain
rules. Think of differentiation as a process during which a function is processed to produce another function
the derivative. This process (differentiation) is carried out using different formulas and rules. We have introduced
the basic ones in this chapter. We have also demonstrated some techniques you can use to compute the derivatives
of different algebraic and trigonometric functions. In addition to this, we have demonstrated how some theorems,
such as the Mean Value Theorem, can be applied.
At this stage you should be able to
compute the derivative of a function from first principles (using the definition of the derivative)
distinguish between the continuity and differentiability of a function
use the basic differentiation formulas to compute derivatives of algebraic and trigonometric functions
compute the derivatives of trigonometric functions and inverse trigonometric functions, as well as exponential
and logarithmic functions
use the methods of logarithmic differentiation and implicit differentiation to find derivatives
solve problems involving tangents and normal lines and apply the Mean Value Theorem
Continue practising solving these problems until you have mastered the basic techniques! Go through the section
For your review at the end of each chapter to consolidate what you have learnt. Also use other textbooks and the
internet.

82

CHAPTER 3
INTEGRATION
1.

Background

In principle, calculus deals with two geometric problems: the one is to find the tangent line to a curve and the other
is one to find the area of a region under a curve. Both these problems are limiting processes. The first one is called
differentiation and the second process is integration. In this chapter we turn to the latter process.
To find the area of the region under the graph of a positive continuous function f defined on an interval [a, b] we
subdivide the interval [a, b] into a finite number of subintervals, say n, the kth subinterval having length xk , and
then we consider the sum of the form nk=1 f (tk ) xk , where tk is some point in the kth subinterval. This sum is an
approximation of the area that is arrived at by adding up n rectangles. Making the subdivisions finer and finer, this
sum will tend to a limit as we let n and, roughly speaking, this limit is the definition of the definite integral
b
a f (x)dx (Riemanns definition).
The two concepts derivative and integral, arise in entirely different ways and it is a remarkable fact indeed
that the two are intimately connected. We will show that differentiation and integration are, in a sense, inverse
operations.
There is a connection between differential calculus and integral calculus. This connection is called the Fundamental
Theorem of Calculus and we shall see in this chapter that it greatly simplifies the solution of many problems.
2.

Learning outcomes

At the end of this chapter, you should be able to


show that you understand the notion of the antiderivative by finding the antiderivative of basic algebraic,
trigonometric, exponential and logarithmic functions,
use the Fundamental Theorem of Calculus to find the derivatives of functions of the form F (x) =

g(x)
a

f (t) dt

83

MAT1512/1

evaluate definite integrals and use them to determine the area between a curve and the x-axis, and the area
between curves
use substitution or term-by-term integration techniques to integrate basic algebraic, trigonometric, exponential
and logarithmic functions
solve problems involving the Mean Value Theorem for definite integrals
Note: The way to master calculus is to solve lots of calculus problems!
3.

Prescribed reading

The overview of chapter 4 of Smith on pages 344 to 428 especially sections 4.1, 4.4, 4.5 and 4.6, as well as chapter
5, section 5.1 and chapter 6, section 6.1.
4.

Worked examples

Our collection of worked examples of the work covered in chapter 4 and sections 5.1 and 6.1 of chapters 5 and 6 of
Smith can be divided into eleven sets (with some overlap), namely:

I. Antiderivatives
II. The definite integral and the Fundamental Theorem of Calculus Part I
III. The definite integral and the area between the curve and the x-axis
IV. The definite integral and the area under the curve
V. The Mean Value Theorem for definite integrals
VI. The Fundamental Theorem of Calculus Part II
VII. Integration in general
VIII. Indefinite integrals
IX. Integration by substitution
X. Integration of exponential and logarithmic functions
XI. Review of formulas and techniques of integration

Attempt the problems appearing immediately after each set once you have studied the relevant parts of Smith and
done some of the exercises there.
The table below (Table 4) shows how these worked examples and their solutions are organised.

84

Table 4
Topic(s)
I.
II.

Antiderivatives

The definite integral and the area


between the curve and the x-axis

IV.

The definite integral and the


area under the curve

110

Section 4.5
pages 383386

1114

Section 5.1

1517

Section 5.1
Section 4.3 (read only)

1821
2223

V.

The Mean Value Theorem for


definite integrals

Section 4.4
page 379

VI.

The Fundamental Theorem of


Calculus Part II

Section 4.5
pages 387388

VIII.
IX.
X.
XI.

Study guide
examples

Section 4.1

The definite integral and the


Fundamental Theorem of
Calculus Part I

III.

VII.

I.

Section in
Smith & Minton

2426
Exercises 2730

Integration in general

Chapter 4 & 6, Section 6.1

1152

Indefinite integrals

Section 4.1, pages 346351

3134

Section 4.6

3537

Integration by substitution
Integration of exponential and
logarithmic functions

Section 4.6, Example 6.11


Section 4.8

Review of formulas and


techniques of integration

Section 6.1
Examples 1.11.5
pages 510513

3839; 43,45,51
4052

Antiderivatives

If we have been given a function, we can find its derivative. But many mathematical problems and their applications
require us to solve the inverse (the reverse) of the derivative problem. This means that, given a function f , you
may be required to find a function F of which the derivative is f . If such a function F exists, it is called the
antiderivative.
The process of undoing differentiation (in other words, the reverse of differentiation) is known as integration. The
antiderivative is known as an integral.
Find the general antiderivatives of the following functions:
1. x 3 + x 2
2.

26x
2

3. x 3 + 2x 3
4. + sec tan

85
3
3
5. cosec2
2
2
6. cos 2x + cos x
Find the function f if f is given as follows:
7. f (x) = sin x

x2

8. f (x) = 3 cos x + 5 sin x

9. f (t) = sin 4t 2 t

7
10. f (t) = t 4 + t 6
Solutions:

1. x 3 + x 2
General antiderivative:

x3
1
+c
x 2 +
2
3

by Rule 4 on page 255 in Smith (the power rule).


2.

26x

= 12 x 6

General antiderivative:

=
2

1 6 5
x 6 + c (power rule)
2 5
3 5
x 6 + c.
5

3. x 3 + 2x 3
General antiderivative:

3 2
3 5
x 3 + 2. x 3 + c (power rule)
5
2
3 5
2
x 3 + 3x 3 + c.
5

4. + sec tan
General antiderivative:

3
3
5. cosec2
2
2
General antiderivative:

1 2
+ sec + c.
2

3
cot + c.
2

MAT1512/1

86
6. cos 2 x + cos x
General antiderivative:

sin x + sin x + c.

7.
f (x) = sin x

= sin x x

x2

2
5

5 7
f (x) = cos x x 5 + c.
7
8.
f (x) = 3 cos x + 5 sin x

f (x) = 3 sin x 5 cos x + c.


9.

1
f (t) = sin 4t 2 t = sin 4t 2t 2
1
2 3
f (t) = cos 4t 2 t 2 + c
4
3
1
4 3
= cos 4t t 2 + c.
4
3
10.

4
7
t 4 + t 6 = t 7 + t 6
7 11 1 5
f (t) =
t 7 t + c.
11
5
f (t) =

II.

The definite integral and the Fundamental Theorem of Calculus Part I

The Fundamental Theorem of Calculus, Part I is stated as follows:

If f is continuous on [a, b] and F (x) is any antiderivative of f (x) , then


b
a f (x) dx = F (b) F (a)
Examples:
Compute the following integrals using the Fundamental Theorem of Calculus:
11.
2
0

x 2 2x dx =
=

2
1 3
x x2
3
0
4
8
4 = .
3
3

87

MAT1512/1

12.
4

1
dx =
x2

x 2 x 2 d x

4
2 3
x 2 + x 1
=
3
1
16 1 2
+ 1=
3
4 3

=
=

2
2
3
+ 11
(4) 2 + 41
3
3
47
.
12

13.
4

e2x dx
0
4
1
1
1
e2x
= e8 e 0
2
2
2
0
1 8 1
= e + .
2
2

Note that in the example above, the integral of an e function is just the e function with its index divided by
the derivative of its index.

14.
2
dx
3 x
= 2 ln |x|1
3 = 2 (ln |1| ln |3|)
1

= 2 (ln 1 ln 3) = 2 ln 3.

III.

The definite integral and the area between the curve and the x-axis

It is important to understand that the definite integral of a continuous function f evaluates the area between this
function f and the x-axis. See the following examples:
Examples:

15. Find the area under the curve f (x) = sin x on the interval [0, ] if sin x 0.
Solution:

Area =

sin xd x = (cos ) (cos 0) = (1) (1) = 2.

16. [The connection between area and calculus]


Calculate the value of
the aid of integration.

5
2 (2x

4) dx geometrically by using a suitable sketch. Then test your answer with

88
Solution:
(a) Geometrically:
The graph of y = 2x 4 looks like this:
y
y =2x

(5, 6)

x
(2, 0)

(5, 0)

(0, _ 4)

Therefore
5
2

(2x 4)dx = area of triangle A


= 1/2 base height
= 1/2 3 6
= 9.

(b) Test with the aid of integration:


F(x) = x 2 4x is an antiderivative of y = 2x 4 and hence
5
2

(2x 4)d x = F(5) F(2)


= (25 20) (4 8)

= 9,
which is the same as in (a) above.

89

MAT1512/1

17. Find the area of the region between the x-axis and the graph of f (x) = 2x + x 2 x 3 , 1 x 2.
Solution:

y = f ( x)

x
1

First find the zeros of f . Since


f (x) = 2x + x 2 x 3 = x(2 x + x 2 ) = x(x + 1)(x 2),
the zeros are x = 0, 1, and 2. It is clear that the zeros partition the interval [1, 2] into two subintervals:[1, 0]
and [0, 2] with f (x) 0 for all x [1, 0] and f (x) 0 for all x [0, 2]. We integrate over each subinterval
and add the absolute values of the calculated values.
Integral over [1, 0]:
0
1

x3
x4 0

3
4 1
(1)3 (1)4
= 0 (1)2 +

3
4
5
= .
12

(2x + x 2 x 3 )dx =

x2 +

Integral over [0, 2]:


2
0

Enclosed area:
Total enclosed area =

5
12

8
3

x3
x4 2

3
4 0
3
(2)
(2)4
= (2)2 +

0
3
4
8
= [4 + 4]
3
8
.
=
3

(2x + x 2 x 3 )dx =

37
.
12

x2 +

90
IV.

The definite integral and the area under the curve

If f and g are continuous functions with f (x) g (x) throughout [a, b], then the area of the region between the
curves of f and g from a to b is the integral of [ f g] from a to b. The area bounded by the two curves y = f (x)
and y = g (x) on the interval [a, b] where f and g are continuous, is written as follows:
b

A=

f (x) g (x) d x.

This is shown in the sketch below:


y

y = f (x)
This is only valid if f (x) g (x) on [a, b] , that is
when f (x) is above g (x) or f (x) is to
the right of g (x) .

a
b

y = g( x )
x

Examples:
18. Evaluate the area between the curves f (x) = 5 x 2 and g (x) = |x + 1|.
Solution:
y
5
y = f ( x)
y = g ( x)

1
_

5
a=

_2

_1

_
b = 1+ 17
2

91

MAT1512/1

Points of intersection:
(x + 1) = 5 x 2

x2 x 6 = 0

(x 3)(x + 2) = 0 x = 3 or x = 2,
and
x + 1 = 5 x2

x2 + x 4 = 0

1 17

x =
.
2
It is clear that the limits of integration are the x-values of the points of intersection of the two curves: a = 2 and

b = 1+2 17 . Now we evaluate the area of the region by integrating:

A =
=
=
=
=
=

1+ 17
2

2
1
2
1
2

[5 x 2 |x + 1|]d x

[5 x 2 (x 1)]d x +
2

[x + x + 6]dx +

x2
x3
+
+ 6x

3
2

1+ 17
2

1+ 17
2

[5 x 2 (x + 1)]d x

[x 2 x + 4]dx

1+ 17
x3
x2

+ 4x 1 2
3
2
3

1 + 17
1 + 17
1 1
8
+ 6
+ 2 12 +
+
3 2
3
24
8
3
2

(1 + 17)
(1 + 17)
8
+9

+ 2(1 + 17)
3
24
8

1
2

+ 4

17
1 1 4
2
3 2

1 +

We would like to encourage you to leave your answers unsimplified, as you are not allowed to use a calculator
in your examinations!

19. Find the area bounded by the graphs of y = 3 x and y = x 2 9.


Solution:
Find the points of intersection by equating the two functions:
3 x = x2 9

that is

0 = x 2 + x 12 = (x 3) (x + 4)

x = 3 or x = 4.

92
The two curves intersect at x = 3 and x = 4 (see figure below).
y
y = x2 _ 9

_4

y =3 _ x

The height of the bounded area is h (x) = (3 x) x 2 9 .


Therefore
3

A =

(3 x) x 2 9 d x

4
3

x 2 x + 12 dx =
3

x3
x2

+ 12x
3
2
3

3
4

3
3
(4)
(4)

+ 12 (3)

+ 12 (4)
3
2
3
2

343
.
6

20. Find the area bounded by the graphs of y = x 2 and y = 2 x 2 for 0 x 2.


Solution:

y
y= x

2
x
1
y = 2 _ x2

93

MAT1512/1

Since the curves intersect in the middle of the interval, you have to compute two integrals, namely for 2 x 2 x 2
and for x 2 2 x 2 .
To find the point of intersection, solve for x :
x 2 = 2 x 2 2x 2 = 2 x = 1.
Note that only x = +1 is relevant since x = 1 is outside the given interval.
1

A =

2 x2 x2 dx +

(2 2x) dx +
1

=
=

1
3

2
1

x2 2 x2

dx

2x 2 2 d x
2

2x 3
2x
+
2x
2x
3 0
3
4 4 4
+ + = 4.
3 3 3

= 2

2
16
2
(0 0) +
4
2
3
3
3

21. Find the area bounded by the graphs of x = y 2 and x = 2 y 2 .


Solution:

2
x = y2
1

x
1
_1

2
x = 2 _ y2

_2
It is easier to compute this area by integrating with respect to y. Therefore we find the points of intersection by
solving for y.
y 2 = 2 y 2 y 2 = 1 y = 1.

94
In the interval [1, 1] ,
1

A =

2
2y y 3
3

=
V.

2 y 2 y 2 dy =
1
1

= 2

1
1

2 2y 2 dy

2
2
2 +
3
3

8
= .
3

The Mean Value Theorem for definite integrals

Now we state the Mean Value Theorem for definite integrals as follows (analogous to theorem 4.4 on p 379 in
Smith):

If f is continuous on the interval [a, b] there is at


least one number c between a and b such that
the area under the curve
=
between a and b
or f (c) =

1
ba

b
a

b
a

f (x) d x = (b a) f (c)

f (x) dx.

22. [Finding c using the Mean Value Theorem for integrals]

Find a value c guaranteed by the Mean Value Theorem for integrals for f (x) = x 2 + 2x + 3 on [0, 2]

Solution:
2
0

x 2 + 2x + 3 dx =
=
=
=

1 3
x + x 2 + 3x
3
8
+4+6
3
8 + 12 + 18
3
38
.
3

2
0

95

MAT1512/1

The region bounded by f and the x-axis on [0, 2] is shaded in the next figure.

y
14
12
10
8
6
4
2
_2

_1

The Mean Value Theorem for integrals asserts the existence of a number c on [0, 2] such that f (c) (b a) =
38
. We solve this equation to find c:
3
38
3
38
2
c + 2c + 3 (2 0) =
3
38
2c2 + 4c + 6 =
3
6c2 + 12c 20 = 0
f (c) (b a) =

3c2 + 6c 10 = 0

6 36 + 4 (3) (10)
c =
6
6 156
=
6
39
= 1
.
3

Since only one of these points, that is c = 1 +


the Mean Value Theorem for integrals.

39
3

1, 08, lies within the interval [0, 2] , this value of c satisfies

The following is the definition of the average value of a function:


If f is integrable on [a, b] , then the average value
or mean value of f on [a, b] denoted by av ( f ) is
b
1
av ( f ) = ba
a f (x) dx

96
23. [Finding the average value of a function on an interval]
Find the average value of f (x) = 2x on the interval [1, 5] .
Solution:
The average value is
5
1
1 2
av ( f ) =
2xd x =
x
51 1
4
1
=
(25 1)
4
= 6.

5
1

Obviously this average value occurs when f (x) = 6 and when x = 3.


VI.

The Fundamental Theorem of Calculus Part II

If f is continuous on [a, b] and F (x) =


In other words, if F (x) =

x
a

x
0

f (t) dt, then F (x) = f (x) on [a, b].

f (t) dt where a is any real number, then


d
d
(F (x)) =
dx
dx

f (t) dt
a

d
that is F (x) = f (x) dx
(x) = f (x) (1); therefore F (x) = f (x) .

To apply Part II of the Fundamental Theorem of Calculus, just leave the function f (t) as it is, but replace the
variable t with the variable x times the derivative of x.
Let us write down the two parts of the Fundamental Theorem of Calculus again:
Part I
If G(x) is any antiderivative of f (x) on [a, b] so that G (x) = f (x), then
b

f (x) dx = G (b) G (a) .

Part II (the Fundamental Theorem of Calculus stated in another way)


Suppose that the function f is continuous on the interval [a, b].
Let the function F be defined on [a, b] by F (x) =

x
a

f (t) dt.

Then F is differentiable on [a, b] and F (x) = f (x), which means that F is an antiderivative of f on [a, b] .
Expressed in symbols:

d
dx

x
a

f (t) dt = f (x) = F (x) .

97

MAT1512/1

Remarks:
(1) You should remember that both conclusions of the Fundamental Theorem are useful. Part II concerns the
derivative of an integral it tells you how to differentiate a definite integral with respect to its upper limit.
Part I concerns the integral of a derivative it tells you how to evaluate a definite integral if you can find an
antiderivative of the integrand.
(2) Remember that differentiation and integration are inverse operations and cancel each other to a certain extent.
(3) Notice that the lower limit of the integral may be any constant in the interval [a, b] and it does not affect the
answer after differentiation. Thus
d
dx

x
0

f (t)dt =

d
dx

x
10

f (t)dt =

d
dx

f (t)dt
513

etc. This is so because the derivative of a constant is always zero.


(4) Note that the upper limit must be the same as the variable with respect to which we differentiate, in other
d
words, if we determine
(the integral) then the upper limit must be x (or a function of x).
dx
(5) The dt in the integral shows you that integration is done with respect to the variable t. Hence you must change
all ts in the given function f (t) to xs to obtain f (x), or d F/d x.
(6) If the upper limit of the integral (which you must differentiate) does not consist of the variable of interest
only, but is in fact a function, then you must remember to use the chain rule for differentiation. Thus:
d
dx

g(x)
a

f (t)dt = f (g(x)).g (x).

In this case you replace all ts by g(x) and then multiply by the derivative of g(x) (look at examples 25 and
26 below).
Keeping the above-mentioned in mind, consider the following examples:
Examples:
Determine the derivatives of F(x) in each of the following:
x

24. F(x) =

1
dt
1 + sin2 t

x3

25. F(x) =

1
dt
1 + sin2 t

cos x

26. F(x) =

sin x

t (5 t)dt

98
Solutions:
x

1
dt
2
a 1 + sin t
It follows from part Ie of the theorem that

24. F(x) =

F (x) =

1
.
1 + sin2 x

x3

1
dt
2
a 1 + sin t
In this case we must apply the chain rule, because the upper limit is a function of x. Let f (t) =
g(x) = x 3 . Then

25. F(x) =

F (x) =
=
=

1
1+sin2 t

and

f (g (x)) g (x)
d 3
f x3
x
dx
1
3x 2 .
1 + sin2 x 3
1
1+sin2 t

(In other words, substitute t in the function f (t) =


derivative of x 3 with this answer.)

with x 3 , but also remember to multiply the

26. In this example see that both the upper limit and lower limits are functions of x, so that it follows from remark
3 above that we may introduce any constant in this case. We call it a.
cos x

F (x) =
=

sin x
a
sin x

t (5 t) dt.

cos x

t (5 t) dt +

t (5 t) dt

(from 10, table 5.1 in Smith)


[from 5, table 5.3 in Smith]
sin x

cos x

t (5 t) dt +

t (5 t) dt

(from 2, table 5.1 in Smith)


[from 2, table 5.3 in Smith].
By again applying the chain rule as in example 25, we obtain
d
d
sin x + cos x (5 cos x)
cos x
dx
dx
= sin x cos x (5 sin x) cos x sin x (5 cos x) .

F (x) = sin x (5 sin x)

Now attempt the following yourselves.

99

MAT1512/1

Find the derivatives of F in each of the following cases:


10

27. F(x) =

t+

1
dt.
t

x3

28. F(x) =

cos t dt.
2x
x3

29. F(x) =
30. F(x) =

sin3 t dt.

3
sin x

1 t 2 dt.

Solution:
1
27. x .
x
10

(This is plainly part II of the Fundamental Theorem of Calculus with


x

28. 3x 2 cos x 3 2 cos 2x.

f (x) dx =
x3

(Again we must introduce constants for the lower limits, for example

f (x) d x.)
10

. . .+
2x

x3

2x

... =

. . .+
0

. . ..)
0

29. 3x 2 sin3 x 3 .
(Use the chain rule, in other words first substitute t with x 3 and then multiply the answer with the derivative
of x 3 .)
30. |cos x| cos x.
VII.

Integration in general

The process of computing an integral (integration) is written as follows:


If F (x) = f (x) , then

that is F (x) =

F (x) d x =

f (x) dx,

f (x) d x.

f (x) is the integrand and the term dx identifies x as the variable of integration.
Given a function f , find, if possible, a function F such that:
F (x) = f (x)
A function F defined on an interval which has the property that F (x) = f (x) is called an indefinite integral or
antiderivative of f on the same interval for all x, and we write:

100

F (x) =

f (x) dx (for indefinite integrals)


b

or

F (x) =

f (x) dx (for definite integrals)


a

The term dx is part of the notation. It represents the statement We integrate with respect to x.
VIII.

Indefinite integrals

For indefinite integrals, if F and G are both antiderivatives of f on the interval I, then G (x) = F (x) + c for some
constant c.
If F is any antiderivative of f , the indefinite integral of f (x) with respect to x is defined by
(where c is the constant of integration).
Example:
27. Evaluate

t 5 dt.

Solution:
We know that

x n dx =

d
d 6
t = 6t 5 , so
dt
dt

x n+1
d
+ c since
n+1
dx

However, we could also have


So, in general,

x n dx =

1 6
t = t 5 . Therefore
6

1
t 5 dt = t 6 + c.
6

1
x n+1 + c = x n .
n+1
x n dx =

x n+1
d
+ 5, because
n+1
dx

x n+1
+ 5 = xn.
n+1

x n+1
+ c, where c is the constant of integration.
n+1

Note: This rule does not work for n = 1 since it would produce a division by 0.
Examples of indefinite integrals:
31.
x 17 d x
=
32.

x 18
x 17+1
+c =
+ c.
17 + 1
18
1
dx

3
x
1

x3 dx =

=
2

x3
2
3

+c =

x 3 +1
+c
13 + 1

3 2
x 3 + c.
2

f (x) d x = F (x)+c

101

MAT1512/1

33.
3 cos x + 4x 8 dx
= 3

cos xd x + 4

= 3 sin x + 4

x 8d x

4
x9
+ c = 3 sin x + x 9 + c.
9
9

34.
2
dx
1 + x2
1
= 3 ex d x 2
dx
1 + x2
= 3e x 2 tan1 x + c.
3e x

IX.

Integration by substitution

(a). Indefinite integrals


The method we are going to discuss now is the principal method by which integrals are evaluated.
When the integrand is not in standard form, first of all determine whether the integrand does not consist of two
factors, one of which is the derivative of the other. The method of substitution is the integration version of the chain
rule.
Remember that, according to the chain rule, the derivative of 3x 2 5x + 2
d
3x 2 5x + 2
dx

is

= 4 3x 2 5x + 2 (6x 5) .

On the right-hand side we now have two expressions, namely


3x 2 5x + 2 and 6x 5, which are such that
d
3x 2 5x + 2 = 6x 5.
dx
Since we know that differentiation and integration are inverse operations, we see that
4 3x 2 5x + 2

(6x 5) dx = 3x 2 5x + 2

Example:
Integration of an indefinite integral by substituion
Find
6

7 x 2 + 3x + 5 (2x + 3) dx.
Solution:

+ c.

102
Look at the problem and make the observation as shown in the boxes:
7

x 2 + 3x + 5

(2x + 3)

dx

The derivative of the function in the left-hand box


which is (2x + 3) d x is sitting in the right-hand box.
So let u = x 2 + 3x + 5.
(NB: Note that the function u appears without the index 6.)
Then

du
= 2x + 3, that is (2x + 3) d x = du.
dx

(This is the form in the right-hand box.)


Now substitute everything under the integral sign to express the integral in terms of u. We have
u 6 du = 7

1 7
u + c = u 7 + c.
7

Then back-substitute to express u in terms of x:


7

u 6 du = x 2 + 3x + 5

+ c.

Remarks:
1. Sometimes the two functions under the integral sign are linked, but a constant might be missing.
If, however, you follow the procedure of substitution correctly, the end product will be correct.
Example:
Find

x 2 + 2x + 5

(x + 1) d x

Solution:
Let
u = x 2 + 2x + 5.
Then

Thus (x + 1) dx =

du
= 2x + 2 = 2 (x + 1) .
dx
du
.
2

Substitution then gives


u4

du
2

=
=
=

1
u 4 du
2
1 5
u +c
10
1 2
x + 2x + 5
10

+ c.

103

MAT1512/1

If you differentiate this using the chain rule, you will obtain the function under the integral sign.
2. After you have done your substitution and simplified, there should be no left-over x-values in the integrand.
Example:
Substitution with leftover x-values
Find
x (3x 5)3 dx
Solution:
In this case it is so that the derivative of the function with index 3, that is (3x 5) , is not linked to the other
function in the ordinary way. However, we follow the same method.
Let u = 3x 5, so du = 3dx and dx =

du
.
3

Now substitute:
x (3x 5)3 d x =

xu 3

du
1
=
3
3

xu 3 du

It is clear that there is a left-over x-value.


We now follow the next procedure to eliminate the left-over x-term:
Because u = 3x 5, we can solve for x :

x=

u+5
,
3

so
1
3

xu 3 du =
=
=
=

Examples:
Determine the following integrals:
35.

3x 1 2x 2 dx

36.

cos3 x sin5 x d x

37.

dx
x2 1 +

1 3
x

1
u+5 3
u du
3
3
1
u 4 + 5u 3 du
9
1 u5 5 4
+ u +c
9 5
4
1
5
(3x 5)5 +
(3x 5)4 + c.
45
36

104
Solutions:
35. We see that x is the derivative (except for a constant factor) of 1 2x 2 . Therefore set
u = 1 2x 2 .
Then
du = 4x d x.
In other words

1
du = x d x.
4

Now
3x 1 2x 2 dx = 3

1 2x 2 x dx

1
u
du
4

= 3
=

3
4

u 2 du
3

3 u2
+c
4 3/2
1 3
= u2 + c
2
1
= 1 2x 2
2

3
2

+ c.

Note: You must not give your answer in terms of u. You must give your answer in terms of x, since that was
the original variable!
36. In this case we use trigonometric identities. We do this with two different methods.
Method 1
cos3 x sin5 x dx

cos x cos2 x

sin5 x

d x.

The part in the block, without its power, has the derivative cos x.
Now we must also rewrite the other extra term in terms of sin x, that is cos2 x = 1 sin2 x.
The integral then becomes
cos x 1 sin2 x sin5 x dx
=

cos x sin5 x dx

Since cos x is the derivative of sin x we let u = sin x.

cos x sin7 x d x.

105

MAT1512/1

Then
du = cos x dx
and so it follows that
cos3 x sin5 x dx =
=
=

u 5 u 7 du
u6 u8

+c
6
8
sin8 x
sin6 x

+ c.
6
8

Method 2
cos3 x sin5 x dx
(cos x)3

sin4 x sin x dx.

The part in the block, without the power, has the derivative ( sin x) (thus the constant (1) is missing). Now
rewrite sin4 x in terms of cos x:
sin4 x = 1 cos2 x

= 1 2 cos2 x + cos4 x.

Thus
cos3 x sin5 x d x =
=

(cos x)3 1 2 cos2 x + cos4 x sin x dx


cos3 x sin x d x 2

cos5 x sin x dx +

cos7 x sin x d x.

Put u = cos x, so that du = ( sin x) dx.


Then
cos3 x sin5 x d x =

u 3 du + 2

u 5 du

u 7 du

2
1
1
= u4 + u6 u8 + c
4
6
8
1
1
1
= (cos x)4 + (cos x)6 (cos x)8 + c.
4
3
8
37. In this case we see that x12 is the derivative of 1 +

1
x

and hence we set

1
u =1+ ,
x
and therefore
du =

1
dx
x2

106
and
dx
x2 1 +

1 3
x

du
u3

u 2
+c
2
1
1
+c
2 1+ 1 2

=
=

1
2

x
x +1

+ c.

(b) Definite integrals


The section on changing the limits of integration on pages 398 to 399 in Smith is very important and students tend
to have problems with this. We shall discuss it in detail and show you three different methods you may use. Work
thoroughly through all three and make very sure that you understand each step.
Example:
Calculate

x3
x2 + 1

Solution:

3
2

dx.

Method 1
When you have done substitution with the variable u, also change the limits of integration in terms of the variable
u, and do not change back to the variable x.
1
0

x3
x2 + 1

3
2

dx :

Let
1

u = (x 2 + 1) 2 .
Then
u2 = x 2 + 1
and
2udu = 2xd x, that is u du = x dx
Now we also change the limits of integration:
If
1

x = 0 then u = (02 + 1) 2 = 1
and if
1

x = 1 then u = (12 + 1) 2 =

2.

107

MAT1512/1

Hence,
1

x3
x2 + 1

3
2

dx =

x 2 x dx

3
2

x2 + 1

u2 1
u du
u3

1
du
u2

1
= u+
u

.
1

Now substitute the values of the limits directly and do not change the variable back to x:
1

x3
x2 + 1

3
2

1
2 + (1 + 1)
2

2
2+
2
=
2
3
2 2.
=
2

dx =

Method 2
With this method we first determine the indefinite integral, change the answer back to the variable x after substitution, and then put in the values of the limits of integration in terms of the variable x.
We first determine

x3
x2 + 1

3
2

d x.

As before, we let u = (x 2 + 1) 2 . Then u du = x d x and


x3
x2 + 1

3
2

dx = u +
=

1
+c
u

x2 + 1

1
2

1
x2 + 1

1
2

+ c.

Now we calculate the definite integral in terms of x:


1
0

x3
x2

+1

3
2

dx =

x2 + 1

1
2

1
x2

+1

1
=
2 + (1 + 1)
2

3
2 2.
=
2

1
2

108
Method 3
With this method we do not change the limits when we make the substitution, but we show clearly in terms of which
variable the limits are given:
1
x3
dx
3
0
x2 + 1 2
1

As before, we let u = (x 2 + 1) 2 . Then u du = x d x and


1
0

x3
x2 + 1

x=1

dx =
3
2

x=0

= u+
=
=

u2 1
u du
u3

1
u

x2 + 1

x=1
x=0
1
2

3
2 2.
2

1
x2 + 1

1
2

Remarks:
1. You must decide for yourself which of these three methods you want to use. Please note the incorrectness of
writing the following:
1
1
1
x3
1 2 du
d
x
=
3
u
0
0
x2 + 1 2
This is wrong, because the limits are not given in terms of the variable u.
The answer you obtain may be correct, but the method is totally wrong. In the examinations or an assignment
you will get no marks for an answer like this.
2. There are only two general methods for integration: integration by parts (which is not part of this module)
and integration by substitution. All other methods boil down to algebraic manipulation, followed by the
application of one of the above-mentioned two methods. Keeping this in mind, you will realise the importance
of this section. Now you must try to do all the exercises in Exercise 4.6.
(c) Steps for Integration by substitution
1. Choose a new variable u (usually the innermost expression).
2. Compute

du
.
dx

3. Replace all terms in the original integrand with an expression involving u and du.
4. Evaluate the resulting (u) integral. (You may have to try a different u if the first one doesnt work.)
5. Replace each occurrence of u in the antiderivative with the corresponding expression in x.

109
X.

MAT1512/1

Integration of exponential and logarithmic functions

We state the following integration formulas again:


1.

ex d x

ex + c

2.

ax d x

ax
+c
ln a

3.

1
dx
x

=
=

ln |x| + c

which has the following forms when we work with functions f (x) .
Following normal substitution, we have
1

e f (x) f (x) d x

a f (x) f (x) d x

a f (x)
+c
ln a

ln | f (x)| + c.

1
f (x) dx
f (x)

e f (x) + c

Now we shall do one more example of integration by substitution to show you how the logarithmic function is used
in this case:
Example:
Determine the following integral:

x2
dx.
1 2x 3

Note that this is of the form 3 above, where f (x) = 1 2x 3 and f (x) = 6x 2 . (Remember that the constant
(6) will only make a difference to the answer as seen in Remark 1 on page 102 of the student guide.)
Solution:
In this case we let
u = 1 2x 3 .
Then
du = 6x 2 dx.
In other words

1
du = x 2 dx.
6

Then
1
du
x2
dx =
3
1 2x
6
u
1
= ln |u| + c
6
1
= ln |1 2x 3 | + c.
6

110
Remark:
When working with exponential and logarithmic functions, never approximate your answers, but please give
exact answers. Consider example 5.6 on page 53 of Smith. The answer to the problem is ln 7 4. This is the exact
answer and if you were to use your pocket calculator to find an approximation of the answer, it would be WRONG.
You need not use the pocket calculator at all when doing your assignments!
Example:
Find the following integrals:
sec x
d x = ln |tan x| + x.
tan x

38.

2x
dx = ln x 2 + 1 + c.
+1

39.

XI.

x2

Note that:

Review of formulas and techniques of integration

Go through the rest of the examples carefully. Evaluate:


40.

3 cos 4xdx

41.

sec 2x tan 2xd x

42.

e32x d x
4

43.

1
3

1+x3

dx

sin x
dx
x

44.

45.

cos xesin x dx

0
0

46.

sin x
dx
cos2 x

3
dx
16 + x 2

47.

x
dx

1 x4

48.

1+x
dx
1 + x2

49.

50.
0

f (x) dx, where f (x) =

x
x 2 +1
x
x 2 +1

if x 1,
if x > 1.

d
x 2 + 1 = 2x
dx

111

MAT1512/1

51.

f (x) dx, where f (x) =

xe x if x < 0,
x 2 e x 3 if x > 0.

2+ x
dx
3 x

52.

Solutions:
1
3
+ c = sin 4x + c.
4
4

40.

3 cos 4xdx = 3 sin 4x

41.

sec 2x tan 2xd x = sec 2x,

42.

e32x d x = e32x ,
4

43.

1
3

1
1
+ c = sec 2x + c.
2
2

1
1
+ c = e32x + c.
2
2

dx :

1+x3

2 1
2
3x 3
dx dx =
du.
Let u = 1 + x then du = x 3 dx =
1
3
2
3x 3
2
3

So

1
3

1+x

2
3

3x 3
3
du = 4
1
2
2
x 3 .u

dx = 4

1
du
u

= 6 ln |u| + c = 6 ln 1 + x 3 + c.

sin x
44.
dx :
x

1
Let u = x then du = dx 2 xdu = 2udu = dx.
2 x

sin u
sin x
So
2udu = 2 sin udu
dx =
u
x

= 2 ( cos u) + c = 2 cos x + c.

45.

cos xesin x dx :

du
.
cos x
We have to change the limits, that is, if x = 0, u = sin 0 = 0 and if x = , u = sin = 0.
Let u = sin x then du = cos xd x dx =
0

cos xe

So

sin x

0
0

dx =

= e e = 1 1 = 0.
0

46.

sin x
dx
cos2 x

du
cos xe .
=
cos x

eu du = [eu ]00

112
0

=
47.

sec x tan xd x = sec x

= sec (0) sec 4

1
1
1
=

cos (0) cos 4


1

=1

1
2

2.

1
3
dx = 3
dx :
2
16 + x
16 + x 2
1
1
x
d x = tan1
+c
Recall:
2
2
a +x
a
a
1
x
1
d x = 3. tan1
+c
3
2
2
4 +x
4
4
x
3
+ c.
= tan1
4
4
x
dx =

1 x4

48.

x
1 x2

dx :

du
.
2x
du
x
1
=

2
2
1u x

Let u = x 2 then du = 2xdx dx =

du
1
x
.
=

2
2
1 u 2x
1
1
= sin1 (u) + c = sin1 x 2 + c.
2
2

So

49.

1+x
1
x
dx =
dx +
dx
2
2
1+x
1+x
1 + x2
1
x
2
=
dx +
. dx
2
2
1+x
1+x 2
1
2x
1
d
x
+
dx
=
1 + x2
2 1 + x2
1
= tan1 x + ln 1 + x 2 + c.
2
2

50.

f (x) d x, where f (x) =

0
2

f (x) d x =

x
dx +
2
x +1

x
x 2 +1
x
x 2 +1
2

if x 1,
if x > 1.

x2
d x.
x2 + 1

du

1 u2

Recall that

f (x)
dx = ln | f (x)| + c
f (x)

113

Note: Rationalise the first integral by


1
2

1
=
2

2x
dx +
2
x +1
1

2x
dx +
2
x +1

x2

1
dx
+1

2
and use long division on the second integral to obtain:
2

1dx

1
dx
+1

x2

1
1
ln x 2 + 1 0 + x]21 arctan x]21
2
1

= ln 2 + 1 + arctan 2.
2
4

xe x if x < 0,
x 2 e x 3 if y > 0.

f (x) d x, where f (x) =

51.
2
2

f (x) d x =
2

2
x2

xe dx +
2

For the first integral,

x 2 e x dx.
0
2

xe x dx :

Let u 1 = x . Then
du 1 = 2xd x and
du 1
.
dx =
2x

For the second integral,

x 2 e x dx :

0
3

Let u 2 = x .
Then du 2 = 3x 2 dx and
du 2
dx = 2 .
3x
So we have
2

xe x d x +
xe

du 1
2x

x 2 e x dx
x 2 eu

du 2
3x 2

1
2

eu du 1 +

1
3

eu du 2

1
1
= eu + c1 + eu + c2 .
2
3
When considering the given definite integral, we ignore the constants c1 and c2 and get:
0
2

xe x dx +
x3

e
=
2

x 2 e x dx

0
3

ex
+
3

MAT1512/1

114
1 2
= ex
2

1 3
+ ex
3
2

2
0

1
1
1 3 1 3
2
= e0 e(2) + e(2) e(0)
2
2
3
3

1
1
1
1
1 e4 e 8 1
+
(1)
(1) e4 + e8 e0 =
2
2
3
3
2
2
3
3
4
8
1e
e 1
=
+
.
2
3

2+ x
52.
dx
3 x

2+ x
x +2
dx
Solution:
dx =

3 x
x +3
This expression is an improper fraction, so we apply long division first.

x +3
x +2

+
x

2+ x
5
So
1 +
dx
dx =
3 x
x +3
1
dx
=
1dx + 5

x +3
1
dx :
Now find

x +3

Let u = x + 3 x = 3 u.
du
1
Then
= and
dx
2 x

dx dx = 2 xdu.
du = 21
x
=

So d x = 2 (3 u) du,
that is dx = (6 + 2u) du.
1
1
So
dx =
(6 + 2u) du

u
x +3
1
2u
= 6
du +
du
u
u
1
du + 2du
= 6
u
= 6 ln |u| + 2u + c.
Therefore,

1
x +2
dx =
1d x + 5
dx

x +3
x +3
= x + 5 (6 ln |u| + 2u + c)
= x 30 ln |u| + 10u + 5c

= x 30 ln 3 x + 10 3 x + k, where C = 5c = k

= x + 10 3 x 30 ln 3 x + k.

115

MAT1512/1

Key points
You should by now have an understanding of the notion of integration as the reverse of differentiation. You should
particularly be able to link it to determining areas between different curves and the x-axis, and the area between
curves. You should now be comfortable with calculating the integrals of several basic algebraic, trigonometric,
exponential and logarithmic functions.
At this stage you should be able to
show that you understand the notion of the antiderivative by finding the antiderivative of basic algebraic,
trigonometric, exponential and logarithmic functions
g(x)

use the Fundamental Theorem of Calculus to find the derivatives of functions of the form F (x) =

f (t) dt
a

evaluate definite integrals and use them to determine the areas between a curve and the x-axis, and the area
between curves
use substitution or term by term integration techniques to integrate basic algebraic, trigonometric, exponential and logarithmic functions
solve problems involving the Mean Value Theorem for integrals
Continue practising solving these problems until you have mastered the basic techniques! Go through the For your
Review section at the end of each chapter to consolidate what you have learnt. Also use other textbooks and the
internet.

116

CHAPTER 4
FIRSTORDER DIFFERENTIAL EQUATIONS,
GROWTH AND DECAY, AND PARTIAL
DERIVATIVES
1.

Background

In this chapter we start by looking at first-order differential equations and then we move on to growth and decay
problems, followed by partial derivatives. In many natural processes, quantities grow and decay at a rate proportional
to their size. This is known as growth and decay. It has applications in chemistry, physics, biology, economics and
many other disciplines. In real life, physical quantities usually depend on two or more variables, so we shall also
turn our attention to functions with several variables and extend the basic ideas of differential calculus to these
functions.
2.

Learning outcomes

At the end of this chapter, you should be able to

solve first-order differential equations with initial values


solve basic reallife problems involving exponential growth and decay
determine the partial derivatives of functions with several variables
Note: The way to master calculus is to solve lots of calculus problems!

117
3.

MAT1512/1

Prescribed reading

In chapter 7 we cover sections 7.1 and 7.2 only (see pages 565 to 584 in Smith). Please also study some sections in
Chapter 12.
4.

Worked examples

The greater part of the collection of worked examples is taken from chapter 7 and chapter 12 of the prescribed
textbook by Smith and Minton. They can be divided into three sets (with some overlap), namely,

I. First-order differential equations


II. Growth and decay
III. Partial derivatives

Attempt the problems appearing immediately after each set, once you have studied the relevant parts of Smith and
done some of the exercises there. The table below (table 5) shows how these worked examples and their solutions
are organised.

Table 5
Topic(s)

Section in
Smith & Minton

Study guide
examples

First-order differential equations

Sections 7.1 & 7.2

1-9

II.

Growth and decay

Sections 7.1 to 7.2

10-20

III.

Partial derivatives

Sections 12.3 & 12.5

21-40

I.

I.

First-order differential equations

An equation that contains an unknown function and some of its derivatives is called a differential equation (DE).
Differential equations are of the type:

or

dy
dx
dy
dx

f (x)

= y.

A first-order differential equation is an equation involving the first derivative.


The general first-order differential equation is of the type:
dy
= g (x, y) ,
dx
where g (x, y) is a function with two variables. We refer to first-order because of the presence of the first derivative
only. In general, a differential equation is one that contains an unknown function and one or more of its derivatives.
The order of the differential equation is the order of the highest derivative that occurs in the equation.

118
(a) The solution of differential equations
A function f is called a solution for a differential equation if the equation is satisfied when y = f (x) and its
derivatives are substituted into the equation. When solving a differential equation, we are expected to find all
possible solutions for the equation.
Example:

1. Solve the differential equation y =

x 2 + 7x + 3
.
y2

Solution:
(i) Separate the variables: y 2 y = x 2 + 7x + 3.

(ii) Integrate both sides with respect to x:

y 2 y (x) d x =
y 2 dy =

x 2 + 7x + 3 d x.

x3
x2
y3
=
+ 7 + 3x + c.
3
3
2

Result of integration:

(iii) Solve for y:

x 2 + 7x + 3 d x or

y=

x3 +

21 2
x
2

+ 9x + 3c.

(b) Separable differential equations


A differential equation is separable if you are able to group the ys with dy and xs with dx. For example, y =
dy
= x y 2 2y .
x y 2 2x y is a separable DE because you can rewrite it as y = x y 2 2y or
dx
Divide both sides by y 2 2y and multiply both sides by d x and obtain:
y2

1
dy = xd x.
2y

On the other hand, an equation like y = x y 2 2x 2 y is not separable.


dy
can be factored as a
A separable equation is a first-order differential equation in which the expression for
dx
function of x times a function of y. That is, it can be written in the form
dy
= g (x) f (y) .
dx
The term separable comes from the fact that the expression on the right side can be separated into a function of
x and a function of y. If f (y) = 0, we could re-write the equation as
g (x)
g (x)
dy
=
=
dx
h (y)
h (y (x))

(1)

119

where h (y) =

MAT1512/1

1
.
f (y)

To solve the equation

g (x)
dy
=
, we rewrite it in the differential form as
dx
h (y)
h (y) dy = g (x) d x,

so that all ys are one side of the equation with dy and all xs are on the other side of the equation with dx. Then
we integrate both sides of the equation:
h (y) dy =

g (x) d x.

This equation defines y as a function of x. In some cases we may be able to solve for y in terms of x.
Note: The justification for the equation above,
follows:
L.H.S. =

h (y) dy =

h (y) dy =

g (x) d x, comes from the substitution rule, as

h (y (x))

dy
dx
dx

g (x)
dx (substituting (1))
h (y (x))

h (y (x))

g (x) dx = R.H.S.

In general, the first-order differential equation is


dy
= h (x, y) .
dx
We consider the special case in which h (x, y) = f (y) g (x) , the product of a function of y alone and a function of
x alone.
Examples are:

and

dy
dx
dy
dx
dy
dx

= x 2 y3,
= e x cos y,
=

The examples below, are not included in this special case:


dy
= x + y;
dx

y
.

1 + x2

dy
= sin (x y) ,
dx

dy
=
dx

1 + x 2 + y2

since in each of these equations the right-handside is not of the form f (y) g (x).
dy
We solve
= f (y) g (x) by first separating the variables, putting all the ys on one side and all the xs on the
dx
other side.
Write
dy
= f (y) g (x)
dx
as
dy
= g (x) d x.
f (y)

120
Then integrate both sides (if you can) to get:
F (y) = G (x) + c,
where F (y) is an antiderivative of

1
and G (x) is an antiderivative of g (x) .
f (y)

If this equation can be solved for y in terms of x, the resulting function is a solution of the differential equation.
Note that in separating the variables, you must assume that f (y) = 0.
If f (y) = 0 for y = y0 , it can be checked that the constant function y (x) = y0 is a solution for the equation.
Examples of general solutions:
dy
= x y.
dx
Solution:

2. Solve

It can be noted that y (x) = 0 is a solution. Now assume y (x) = 0 and separate the variables:
dy
y
dy
y

= x dx
x dx

1 2
x +c
2

ln |y| =

eln|y| = e 2 +c

x2
x2

or

|y| = e 2 ec
|y| = ke

x2
2

where k = ec is a positive constant.

x2

If y 0, then |y| = y = ke 2 .

x2

If y < 0, then |y| = y = +ke 2 .


Therefore,
x2

y=

ke 2
ke

x2
2

if y 0,
if y < 0.

This is equivalent to:


x2

y = ae 2
where a is any constant.
Note that the solution y (x) = 0 is included.

121

MAT1512/1

To check if the answer obtained is the solution to the original differential equation, just differentiate this
answer and you should obtain the original differential equation, namely:
x2

y = ae 2
dy
d
x2
ae 2
=
dx
dx

x2

= ae 2

1
2x
2

x2

= axe 2

x2

= x ae 2
= xy

3. Solve

(Proved).

dy
2x (y 1)
=
.
dx
x2 + 1

Solution:
Note that y (x) = 1 is a solution.
Now assume that y (x) = 1 and separate the variables:
2x
dy
=
dx
2
y1
x +1
1
2x
dy =
dx
2
y1
x +1
ln |y 1| = ln x 2 + 1 + c
2
eln|y1| = eln(x +1)+c

|y 1| =

x 2 + 1 ec = k x 2 + 1 , where k = ec .

Therefore
y 1 = k x 2 + 1
where k is a positive constant.
The answer may be written as:
y = 1 + a x 2 + 1 = 1 + ax 2 + a = ax 2 + (a + 1)
where a is an arbitrary constant and a = k. This answer includes the special solution y (x) = 1.
Check:
dy
dx
dy

dx

= 2ax + 0
= 2x

y1
x2 + 1

where a =

y1
x2 + 1

since

y = 1 + a x2 + 1 .

122
(c) Initial-value problems (IVPs)
In many physical (real-world) problems, we need to find the particular solution that satisfies a condition of the form
y (t0 ) = y0 . This is called the initial condition. The problem of finding a solution for the differential equation
that satisfies the initial condition is called the initial-value problem. In such problems, you are asked to find the
solution y = y (x) or y = f (x) for the differential equation
dy
= g (x, y) ,
dx
the graph of which passes through a given point (a, b), that is, which satisfies the initial condition
y (a) = b.
Examples:
4. Find a solution for the initial-value problem

dy
= x 2 , with initial condition y (3) = +1.
dx

Solution:
There are two steps to follow:
(i) Solve the differential equation by integrating both sides:

dy
= x2
dx
dy = x 2 d x

dy =

y =

or

y (x) =

x 2 dx

1 3
x +c
3
1 3
x + c. This is the general solution for the DE.
3

(ii) We find the constant c by substituting the initial condition y (3) = 1 :


1
(3)3 + c
3
1 = 9 + c
1 =

c = 10.

Therefore, the particular solution to the given initial-value problem is


y (x) =

1 3
x + 10.
3

123

5. Find a solution for the differential equation

MAT1512/1

dy
= 8x 3 + 1 with initial conditon y (1) = 1.
dx

Solution:
(i)

dy
= 8x 3 + 1 (separate dx from dy and integrate)
dx
dy = 8x 3 + 1 dx

dy =

8x 3 + 1 d x

8x 4
+x +c
4
or y (x) = 2x 4 + x + c
y =

(ii) Evaluate the value of c:


y (x) = 2x 4 + x + c

1 = 2 (1)4 + 1 + c (initial condition y (1) = 1)


1 = 2 + 1 + c

c = 1 2 1

c = 4.

The particular solution to the given initial-value problem is


y = 2x 4 + x 4.
See examples 1.1, 1.2 and 1.3 on pages 567 to 570 in Smith.
Other initial-value problems:
6. Solve the initial-value problem

dy
2x
=
, y (1) = 0.
dx
3y 1

Solution:

(3y 1) dy =

2x d x

3 2
y y =
2

2x 2
+ c.
2

Now substitute the given initial values and obtain:


3
(0)2 0 = (1)2 + c
2

0 = 1+c
Thus
or

c = 1.
3 2
y y = x2 1
2
3y 2 2y = 2x 2 2 is the particular solution.

124
7. Solve the initial-value problem

dy
= x 2 y2,
dx

y (0) = b.

Solution:
Note that y (x) = 0 is the solution if b = 0
Assume y = 0 and separate the variables:

dy
= x 2 y2,
dx
dy
= x 2d x
y2
y 2 dy = x 2 d x

y 2 dy =

y 1
1
1

=
=

y (0) = b

x 2 dx

x3
+c
3
1 3
x + c.
3

Substitute the initial values:


1
y
1

b
1

=
=

1 3
x +c
3
1
(0)3 + c
3

= 0+c

1
c = .
b

Therefore
1
x3 1
=

y
3
b
3
1
bx 3

=
y
3b
1 (3b) = y bx 3 3

3b = y bx 3 3 = y bx 3 + 3
3b
.

y=
3 bx 3

The solution includes y (x) = 0.


Note: In initial-value problems (IVPs), you are given initial conditions (in other words, the x and y values), and you
use them in order to evaluate a particular value of a constant c.

125

MAT1512/1

More examples:
8. Solving a logistic growth problem: Given a maximum sustainable population of M = 1 000 and a growth
rate of k = 0.007, find an expression for the population at any time t, given an initial population of y(0) = 350
and assuming logistic growth.

Solution:
Using the solution for a logistic equation, we have k M = 7 and

A = constant,

k Mt
AMe
M = carrying capacity,
y=
, where
k
Mt

1 + Ae
k = growth rate and

t = time.

From the initial conditions, we have 350 = y (0) =


and solving for A, we obtain A =

35
65

which gives the solution of the IVP as y =

1 000A
1+ A

35 000e7t
.
65 + 35e7t

9. Investment strategies for making a million: Money is invested at 8% interest, compounded continuously. If
deposits are made at a rate of R2 000 per year, find the size of the initial investment needed to reach R1
million in 20 years.

Solution:
The constant of integration c is obtained by setting t = 0 and taking A (0) = x,
that is 12.5 ln |0.08x + 2000| = c
so that 12.5 ln |0.08A + 2000| = t + 12.5 ln |0.08x + 2000| .
Now find the value of x such that A (20) = 1 000 000.
12.5 ln |0.08 (1 000 000) + 2000| = 20 + 12.5 ln |0.08x + 2000|
12.5 ln |82 000| 20
= ln |0.08x + 2000|
or
12.5
Solve for x by taking the exponential of both sides:
e(12.5 ln 82 00020)/12.5 = 0.08x + 2000

x=

eln 820001.6 2000


181, 943.93.
0.08

126
II.

Growth and decay

dy
y (t) = Aekt is the general solution for the differential equation y (t) =
= ky (t) .
dt
For k > 0 the equation reflects an exponential growth law and for k < 0, the equation reflects an exponential
decay law.

k > 0 is the growth curve

k < 0 is the decay curve

Exponential growth or decay means that:


y (t) =

dy
= ky (t)
dt

... (1)

Since equation (1) involves the derivative of an unknown function, we call it a differential equation (DE). The aim
is to solve this DE. that is, to find the function y(t).
Recall how to solve separable differential equations. We have
dy
= ky (t) ... (2)
dt
This is a separable DE, that is

dy
= kdt.
y (t)

Integrating both sides of equation (2) with respect to t, we obtain


1
dy =
y (t)

kdt

ln |y| + c1 = kt + c2

or ln |y| = kt + C (where C = c2 c1 ).
Since y (t) > 0, we have ln y (t) = kt + C
and taking exponentials of both sides , we get eln y(t) = ekt+C ,
that is, y (t) = ekt .eC .
Therefore y (t) = Aekt (where A = eC ).

127

MAT1512/1

Examples:
10. A bacterial culture contains 100 cells at a certain point in time. Sixty minutes later, there are 450 cells.
Assuming exponential growth, determine the number of cells present at time t and find the doubling time.
Solution:
Exponential growth means that y (t) = ky (t) and after solving this equation, we have y (t) = Aekt .
At t = 0, y (0) = 100 (initial conditions), we have 100 = y (0) = Ae0 = A.
Then y (t) = 100ekt .
At t = 60 minutes, we have 450 = y (60) = 100e60k .
Solve for k by taking natural logarithms of both sides and obtain
ln 4.5 = ln e60k = 60k,
ln 4.5
so that k =
.
60
Therefore, at any time t we have y (t) = 100ekt = 100 exp

At double the time, y (t) = 2y (0) = 200,


ln 4.5
so that 200 = y (t) = 100 exp
t .
60
ln 4.5
t.
Solve for it and get ln 2 =
60

ln 4.5
t .
60

11. A cup of coffee is 180 F when poured. After 2 minutes in a room at 70 F, the coffee has cooled to 165 F.
Find the temperature at time t and find the time at which the coffee will have cooled to 120 F.
Solution:
The differential equation is y (t) = k y (t) + 70 .
Solving this differential equation, we obtain y (t) = Aekt + 70.
Initial conditions are : 180 = y (0) = Ae0 + 70 = A + 70.
This gives y (0) = 180,
so that A = 110 and y (t) = 110ekt + 70.
We use the second measured temperature (165 F) to solve for k :
165 = y (2) = 110e2k + 70.
Solve for k by subtracting 70 from both sides and dividing by 110 to obtain
e2k =

165 70
95
=
.
110
110

95
95
and, therefore k = 12 ln
.
110
110
When the temperature T = 120, we have 120 = y (t) = 110ekt + 70.
1
5
Solve for t and obtain t = ln .
k 11
Taking natural logarithms of both sides, we get 2k = ln

128
12. Compound interest and an example of decay. Suppose the value of a R10 000 asset decreases continuously
at a constant rate of 24% per year. Find its worth after
(a) 10 years
(b) 20 years
Solution:
The value v(t) of any quantity that is changing at a constant rate r satisfies the equation: v = rv.
Here r = 0.24, so that v (t) = Ae0,24t .
Since the initial asset value is R10 000, we have v (t) = 10 000e0.24t .
We now have 10 000 = v (0) = Ae0 = A.
At time t = 10, the asset value is R10 0000.24(10) R907.18 and
at time t = 20, the asset value is R10 0000.24(20) R82.30.
More worked examples:
13. A bacterial culture starts with 1000 bacteria and after 2 hours there are 2500 bacteria. Assuming that the
culture grows at a rate proportional to its size, find the population after 6 hours.
14. The earths atmospheric pressure p is often modeled by assuming that the rate d p/dh at which p changes
with altitude h above sea level is proportional to p. Suppose that the pressure at sea level is 1 013 millibars
(about 14.7 pounds per square inch) and that the pressure at an altitude of 20 km is 90 millibars.
(a) Solve the initial value problem
Differential equation: dp/dh = kp (k is a constant).

Initial condition: p = p0 when h = 0,

Express p in terms of h. Determine the values of p0 and k from the given altitude-and pressure data.

(b) What is the atmospheric pressure at h = 50 km?

(c) At what altitude does the pressure equal 900 millibars?

15. In some chemical reactions, the rate at which the amount of a substance changes with time is proportional to
the amount present. For the change of gluconolactone into gluconic acid, for example,
dy
= 0.6y
dt
when t is measured in hours. If there are 100 grams of gluconolactone present when t = 0, how many
grams will be left after the first hour?
16. The intensity L (x) of light x feet beneath the surface of the ocean satisfies the differential equation
dL
= k L .
dx
As a diver, you know from experience that diving to 18 ft in the Caribbean Sea cuts the intensity in half. You
cannot work without artificial light when the intensity falls below one-tenth of the surface value. Up to about
how deep can you expect to work without artificial light?

129

MAT1512/1

17. If R1000 is invested for 4 years at 12% interest compounded continuously, what is the value of the investment
at the end of the 4 years?
18. You have just placed A0 rands in a bank account that pays 14% interest, compounded continuously.
(a) How much money will you have in the account in 6 years time?
(b) How long will it take for your money to double and to triple?
19. Suppose an object takes 40 min to cool down from 30 C to 24 C in a room that is kept at 20 C.
(a) What would the temperature of the object be 15 min after it has been 30 C?
(b) How long will it take for the object to cool down to 21 C?
20. Suppose that a cup of soup cooled down from 90 C to 60 C within 10 minutes in a room of which the
temperature was 20 C. Use Newtons Law of Cooling to answer the following questions.
(a) How much longer would it take the soup to cool to 35 C?
(b) Instead of being left to stand in the room, the cup of 90 C soup is put in a freezer in which the temperature is 15 C. How long will it take the soup to cool down from 90 C to 35 C?
Solutions:

13. Let y (t) be the number of bacteria after t hours. Then y0 = y (0) = 1000 and y (2) = 2500. Since we are
dy
assuming
= ky, (Law of Exponential Change on page 380 in Smith),
dt
y (t) = y0 ekt = 1000ekt

y (2) = 1000e2k = 2500.

Therefore e2k = 2.5 and 2k = ln 2.5.


ln 2.5
Thus k =
.
2
Substituting the value of k = 12 ln 2.5 back into the expression for y (t), we have
ln 2, 5
t
y (t) = 1000e 2
= 1000 eln 2.5
t

= 1000 (2.5) 2

t
2

(since eln 2.5 = 2.5).

Therefore the population after 6 hours is


y (6) = 1000 (2.5)3 = 15 625.

130

14.

dp
= kp (k is a constant).
dh
Initial condition: p = p0 when h = 0.

(a) Differential equation:

We separate the variables by dividing the differential equation by p to have


1 dp
= k
p dh
ln | p| = kh + c (by integrating the RHS will respect to h)
| p| = ekh + c (by exponentiating)

| p| = ec ekh

p = ec ekh

p = Aekh where A = ec .
But the initial condition p = p0 when h = 0. Therefore p0 = Aek0 = A and thus
p = p0 ekh
Then
p (0) = p0 = 1013 and p (20) = 90 = 1013e20k .
Thus
90
1013
90
20k = ln
1013
90
1
ln
.
k =
20 1013
e20k =

Therefore p (h) is given by


h
90
= 1013
20
1013

90
p (h) = 1013e ln
1013

h
20

(b) If h = 50 km, then


p (50) = 1013e

ln

= 1013 e

90
1013

ln

52
5
2

90
1013

90
= 1013
1013
902.5
.
=
10131.5

5
2

(since e

ln

90
1013

90
)
1013

131

MAT1512/1

(c) If p = 900, then


90
900 = 1013
1013

h
20

90 20

1013
h
90

ln
20
1013

900
1013
900
= ln
1013
900
1
.
ln
h = 20
90
1013
ln 1013

15. We have the following initial-value problem:


dy
Differential equation:
= 0, 6y.
dt
Initial condition: y0 = 100 when t = 0.

We use the Radioactive decay equation on page 382 in Smith:


y = y0 ekt .

Since y0 = 100 and k = 0, 6 we have

y = 100e0,6t .

After the first hour the number of grams will be


y = 100e(0,6)(1)
= 100e0,6 .

16. We have the following initial-value problem:


dL
Differential equation:
= k L
dx
Initial condition: L = L 0 when x = 0. We are given that L = 12 L 0 when x = 18. We want to find x when
1
L 0.
L = 10
We use equation (1.4) on page 567 in Smith:

L (x) = L 0 ekx
where
L (x) = y (t) ,
L0 =

A.

Thus
1
L 0 = L 0 e18k
2
1
e18k =
2
e18k = 2

18k = ln 2
1
k =
ln 2.
18

132
When L =

1
L
10 0

we have
1
L 0 = L 0 ekx
10
1
ekx =
10
1
kx = ln
10
kx = ln 10
1
x =
ln 10
k
1
18
ln 10 (since k =
ln 2)
=
ln 2
18

17. We use the Continuous compound interest formula on page 381 in Smith:
A (t) = A0 ert .
It is given that A0 =R1000 and r = 12% = 0, 12. So the value of the investment in 4 years will be
A (4) = 1000e(0,12)4
= 1000e0,48 .

18. We use the continuous compound interest formula:


A (t) = A0 ert .
(a) We are given that r = 14% = 0, 14. In 6 years the money will be
A0 e(0,14)6

A (6) =

A0 e0,84 .

(b) We want to find t when A = 2A0 , and when A = 3A0 . We have A (t) = A0 e0,14t .
When A = 2A0 we have

2A0 =

e0,14t

A0 e0,14t

= 2

0, 14t = ln 2
ln 2
100
t =
=
ln 2.
0, 14
14
When A = 3A0 we have
3A0 =

e0,14t

A0 e0,14t

= 3

0, 14t = ln 3
ln 3
100
t =
=
ln 3.
0, 14
14

133

MAT1512/1

19. We use Newtons Law of Cooling on page 569 in Smith:


= Ts + (T0 Ts ) ekt

T
where T

= y (t) , Ts = T

and T0 Ts = A

where T is the temperature at any given time t, Ts is the surrounding temperature and T0 is the initial temperature (in other words, T0 is the value of T at t = 0).
Now, we are given that Ts = 20 C and T0 = 30 C.

Then the objects temperature after t minutes is


T

= 20 + (30 20) ekt

= 20 + 10ekt .

(a) We first find k, by using the information that T = 24 when t = 40 :


24 = 20 + 10e40k

10e40k = 4
4
e40k =
10

4
10

40k = ln
k =

1
4
.
ln
40
10

Now the objects temperature after time t is


1

T = 20 + 10e 40 ln 10 t.
We now find T when t = 15 :

= 20 + 10e 40 ln 10 15

= 20 + 10 eln 10
4
= 20 + 10
10

15
40

15
40

(b) We now find the time t when T = 21 C:


4

21 = 20 + 10 eln 10
4
10
10

4
10

t
4
ln
40
10

t
40

= 1

t
40

1
10

= ln

t =

ln

1
10
40
4
10

ln

40 ln 10
.
ln (0, 4)

1
10

t
40

134
20. As in the solution of the question above,
T = Ts + (T0 Ts ) ekt .
It is given that Ts = 20 C and T0 = 90 C. The soups temperature after t minutes is
T

= 20 + (90 20) ekt

= 20 + 70ekt .

(a) We find k by using the information that T = 60 C when t = 10 minutes:


60 = 20 + 70e10k

40 = 70e10k
4
e10k =
7
4
10k = ln
7
1
4
.
k = ln
10
7
The soups temperature at time t is
T = 20 + 70e

1
10

ln

4
7

We now find the time t when T = 35 C:


4

35 = 20 + 70 eln 7
4
15 = 70
7

4
7

t
4
ln
10
7

t
10

t
10

15
70

15
70
1
15
t = 10
ln .
4
70
ln 7
= ln

(b) With T = 15 C and T0 = 90 C, we have


T

= 15 + (90 (15)) ekt


= 15 + 105ekt .

t
10

135

MAT1512/1

So we find t when T = 35 C:
35 = 15 + 105 e
4
50 = 105
7

III.

4
7

t
4
ln
10
7

t
10

ln

4
7

t
10

t
10

50
105

50
105
1
50
t = 10
.
ln
4
105
ln 7
= ln

Partial derivatives

Functions with more than one variable appear more often in science than functions with one variable. The reason
for this is that in real life the mathematical application in respect of any dynamic phenomenon depends on more
than one variable. One example of a dynamic phenomenon is a moving train, where the motion will depend on
variables such as time, distance, friction, mass and so forth. The applied mathematician uses functions with several
variables, their derivatives and their integrals to study continuum mechanics, probability, statistics, fluid mechanics
and electricity to mention but a few examples.
If f is a function of two independent variables, we usually call the independent variables x and y and picture the
domain of f (x, y) as a region in the XY -plain. If f is a function of three independent variables, we call the variables
x, y, and z and picture the domain of f (x, y, z) as a region in space.
Now, to talk about the derivative of f doesnt make sense if we do not specify with respect to which independent
variable the derivative is required. When we keep all but one of the independent variables of a function constant and
differentiate with respect to that one variable, we get a partial derivative. We would like to encourage you to work
through examples 3.13.6 in section 12.3 and to study section 12.5 of chapter 12 in Smith carefully, in conjunction
with the examples in the study guide.
(a) The partial derivatives technique
This technique is used when functions have more than one variable.
Consider a function f of two variables, x and y. Suppose we let only x vary, while keeping y fixed and y = b
where b is a constant.
Then this is a function of a single variable x : g (x) = f (x, b).
If g has a derivative at a, this is called the partial derivative of f with respect to x at (a, b), denoted by
g (a + h) g (a)
f (a + h, b) f (a, b)
f x (a, b) = g (x) , that is g (a) = lim
or f x (a, b) = lim
h0
h0
h
h
Similarly, the partial derivative of f with respect to y at (a, b), denoted by f y (a, b) is obtained by keeping x fixed
(that is x = a) and finding the derivative at b of the function G (y) = f (a, y).

136

So f x (a, b) = lim

h0

f (a + h, b) f (a, b)
f (a, b + h) f (a, b)
and f y (a, b) = lim
.
h0
h
h

If we now let the point (a, b) vary, f x and f y become functions of two variables.
Definition
If f is a function of two variables, its partial derivatives are the functions f x and f y defined as:
f (x + h, y) f (x, y)
f x (x, y) = lim
h0
h
and
f (x, y + h) f (x, y)
f y (x, y) = lim
h0
h
(b) Notations for partial derivatives
If z = f (x, y) , we write:
f x (x, y) = f x =

z
f
=
f (x, y) =
= f 1 = D1 f = Dx f
x
x
x

f y (x, y) = f y =

z
=
f (x, y) =
= f 2 = D2 f = D y f
y
y
y

Note: f 1 or D1 f indicates differentiation with respect to the first variable and f 2 or D2 f indicates differentiation
with respect to the second variable.
(c) Rules for finding the partial derivatives of z = f (x, y)
1. To find f x , regard y as a constant and differentiate with respect to x.
2. To find f y , regard x as a constant and differentiate with respect to y.

Examples:

21. If f (x, y) = x 3 + x 2 y 3 2y 2 , find f x (2, 1) and f y (2, 1).


22. Find

z
z
and
if z is defined implicitly as a function of x and y by the equation x 3 + y 3 + z 3 + 6x yz = 1.
x
y

Solutions:
21. If f (x, y) = x 3 + x 2 y 3 2y 2 ,
then holding y constant and differentiating with respect to x, we get
f x (x, y) = 3x 2 + 2x y 3 0
= 3x 2 + 2x y 3

137

MAT1512/1

and
f x (2, 1) = 3 (2)2 + 2 (2) 13
= 12 + 4

= 16.

Holding x constant and differentiating with respect to y, we get


f y (x, y) = 0 + 3x 2 y 2 4y
= 3x 2 y 2 4y

and
f y (2, 1) = 3 22

= 12 4

12 4 (1)

= 8.
22. Given that x 3 + y 3 + z 3 + 6x yz = 1.

Then, holding y constant and differentiating implicitly with respect to x, we get


3x 2 + 0 + 3z 2
Solving this equation for

z
z
+ 6yz + 6x y
= 0.
x
x

z
, we get
x
z
x

3 x 2 + 2yz
3x 2 6yz
=
3z 2 + 6yz
3 z 2 + 2yz

x 2 + 2yz
.
z 2 + 2yz

Similarly, holding x constant and implicit differentiation with respect to y gives


z
z
+ 6x z + 6x y
y
y
z
3z 2 + 6x y
y
z

0 + 3y 2 + 3z 2

= 0
= 3y 2 6x z
=
=

3y 2 6x z
3z 2 + 6x y
3 y 2 + 2x z
3 z 2 + 2x y

y 2 + 2x z
.
z 2 + 2x y

138
(d) Partial differentiation
If z = f (x, y) where f is a differentiable function of x and y where x = x (s, t) and y = y (s, t) , and both have
partial derivatives, then the chain rule below holds:
z
s
z
t

=
=

z
x
z
x

x
z y
+
and
s
z s
x
z y
+
.
t
y t

Example:
23. Suppose f (x, y) = e x y , x (u, v) = 3u sin v and y (u, v) = 4v 2 u.
g
g
For g (u, v) = f (x (u, v) , y (u, v)) , find the parial derivatives
and
.
u
v
Solution:
f (x, y) = e x y , x (u, v) = 3u sin v, y (u, v) = 4uv 2 and
g (u, v) = f (x (u, v) , y (u, v)) .
Then

g
f x
f y
=
+
u
x u
y u

and

g
f x
f y
=
+
.
v
x v
y v

So
f
x
f
y
x
u
y
u
x
u
y
u

= e x y y = ye x y ,
= e x y x = xe x y ,
= 3 sin v,
= 4v 2 ,
= 3u ( cos u (1)) = 3u cos v and
= 8uv.

Therefore
g
u

= ye x y (3 sin v) + xe x y 4v 2
2
2
= 4uv 2 e3u sin v (4uv ) (3 sin v) + 3u sin ve3u sin v (4uv ) 4v 2

= 12uv 2 e12u

2 v 2 sin v

2 12u 2 v 2 sin v

= 24uv e

sin v + 12uv 2 e12u


sin v and

2 u 2 sin v

sin v

139
g
v

MAT1512/1

= ye x y (3u cos v) + xe x y (8uv) = 3yue x y cos v + 8xuve x y

2
2
= 3 4uv 2 ue3u sin v (4uv ) cos v + 8 (3u sin v) uve3u sin v (4uv )

= 12u 2 v 2 e12u

= 24uv 2 e

2 v 2 sin v

12u 2 v 2 sin v

cos v + 24u 2 ve12u

2 v 2 sin v

sin v

sin v.

(e) Functions of more than two variables


Partial derivatives can also be defined for functions of three or more variables . For example, if f is a function of
three variables, x, y and z, then its partial derivative with respect to x is defined as:
f x (x, y, z) = lim

h0

f (x + h, y, z) f (x, y, z)
h

and is found by, for example, regarding y and z as constants and differentiating f (x, y, z) with respect to x.
Example:
24. Find f x , f y and f z if f (x, y, z) = e x y ln z.
Solution:
If f (x, y, z) = e x y ln z,
then to compute the partial derivative with respect to x, we treat y and z as constants and obtain
fx

e x y ln z
x
= e x y (y) ln z
=

= ye x y ln z.

To compute the partial derivative with respect to y, we treat x and z as constants and obtain
xy
e ln z
y
= e x y (x) ln z

fy =

= xe x y ln z.

To compute the partical derivative with respect to z, we treat x and y as constants and obtain
xy
e ln z
z
1
= e x y (1)
z
ex y
=
.
z

fz =

140
(f) Higherorder derivatives
If f is a function of two variables, then its partial derivatives f x and f y are also functions of two variables, so we
can consider their partial derivatives ( f x )x , ( f x ) y , f y x and f y y , which are called the second partial derivatives
of f .
If z = f (x, y) , we use the following notation:
( f x )x

( f x )y =

= f 12 =
y

= f 21 =
x

= f 22 =
y

f x x = f 11 =
fx y

fy

f yx

fy

f yy

f
x
f
x
f
y
f
y

2 f
2z
=
,
x 2
x 2
2 f
2z
=
=
,
y x
y x
2 f
2z
=
=
and
xy
xy
2 f
2z
=
=
.
y 2
y2
=

2 f
means we first differentiate with respect to x and then with respect to y, and in
y x
computing f yx , the order of differentiation is reversed.

Note: The notation f x y or

Example:
25. Find the second partial derivatives of f (x, y) = x 3 + x 2 y 3 2y 2 .
Solution:
Given f (x, y) = x 3 + x 2 y 3 2y.
In example 21 we found that
f x (x, y) = 3x 2 + 2x y 3 and
f y (x, y) = 3x 2 y 2 4y.
Therefore

3x 2 + 2x y 3 = 6x + 2y 3 ,
f x x (x, y) =
x

3x 2 + 2x y 3 = 0 + 6x y 2 = 6x y 2 ,
f x y (x, y) =
y

3x 2 y 2 4y = 6x y 2 0 = 6x y 2 and
f yx (x, y) =
x

3x 2 y 2 4y = 6x 2 y 4.
f yy (x, y) =
y
Now work through the rest of the examples provided.
Worked examples:

26. Determine f / x and f / y if:


(a) f (x, y) = sin(x + y) and

141
(b) f (x, y) =

x2

MAT1512/1

x
.
+ y2

27. Use the chain rule for partial derivatives to determine the value of dw/dt when t = 0 if w = x 2 + y 2 ,
x = cos t + sin t and y = cos t sin t.
28. Use the chain rule for partial derivatives to find dw/dt when w = sin (x y)
x = 1 + t and y = t 2 .
29. Suppose
w=

1
1
+ , x = t 2 + 1 and y = t 4 + 3.
xy
y

Use the chain rule for partial derivatives to find dw/dt.


30. Assume that the equation
4

yx 3 + y 3 = 144
defines y as a differentiable function of x. Use partial derivatives to find the value of dy/d x at the point
(8, 8).
31. Determine z/u when u = 0 and v = 1, if z = sin(x y) + x sin y, x = u 2 + v 2 and y = uv.
32. Let g be a function of x and y, and suppose that x and y are both functions of u and v. Which one of the
following is g/v?
g
u
dg
(b)
dx
g
(c)
x
dg
(d)
dx
(a)

u
x
x

v
x

v
dx

dv

g u

u y
dg y
+

dy v
g y
+

y v
dg dy
+

dy dv

33. Determine 2 f /y 2 if f (x, y) = tan(x y) cot x.


34. Suppose that the equation
y = sin(x y)
defines y as a differentiable function of x. Use partial derivatives to find dy/dx.
35. Suppose w = (2x 2 + 2y 2 2)2 , x = 2u 2v + 2 and y = u + 4v + 2. Find the value of w/v when
u = 0 and v = 1.
36. Use the chain rule for partial derivatives to find dw/dt when w = sin(x y), x = 1 + t and y = t 2 .
37. Assume that the equation
x y + y 2 3x 3 = 0
defines y as a differentiable function of x. Find the value of dy/d x at the point (1, 1) by using partial
derivatives.

142
38. Use the chain rule for partial derivatives to find w/r when r = and s = 0, if w = sin(2x y),
x = r + sin s and y = rs.
39. Suppose that the equation
7y 4 + x 3 y + x = 4
defines y as a differentiable function of x. Find the value of dy/dx at the point (4, 0).
40. The dimensions a, b and c of a rectangular solid vary with time (t). At the instant in question
dc
da
=
= 2 cm/sec
dt
dt

a = 13 cm, b = 9 cm, c = 5 cm,


and

db
= 5 cm/sec.
dt

diagonal
c

b
a
(a) How fast is the volume V changing at the given instant? Is V increasing or decreasing?
(b) How fast is the surface area S changing at that moment? Is the area increasing or decreasing?
(c) How fast is the length D of the diagonal changing at that instant? Does the length increase or decrease?

Solutions:
26.

(a)
f (x, y) = sin(x + y)
f
f
= cos(x + y) and
= cos(x + y).

x
y
(b)
f (x, y) =

f
x

=
=

x
x 2 + y2
1. x 2 + y 2 x (2x + 0)
x 2 + y2

y2 x 2

x 2 + y2

143

MAT1512/1

Also,
f (x, y) =

f
y

x2

x
= x(x 2 + y 2 )1
+ y2

= x((x 2 + y 2 )2 ).(0 + 2y)


=

2x y
x2

+ y2

27.
w = x 2 + y 2 , x = cos t + sin t and y = cos t sin t
w d x
w dy
dw
=
+

dt
x dt
y dt
= (2x + 0)( sin t + cos t) + (0 + 2y)( sin t cos t)
= 2x (cos t sin t) 2y (sin t + cos t)
by substituting for x and y

= 2(cos t + sin t)(cos t sin t) 2(cos t sin t)(cos t + sin t)

= 0.
Hence,

dw
dt

t=0

= 0.

28.
w = sin (x y)

and

w
= cos (x y) y
x

w
= cos (x y) x.
y

Also,

x = 1+t
dx
= 1,
dt

and

y = t2
dy
= 2t.
dt

We have
dw
dt

w dx
w dy

x dt
y dt
= cos (x y) y (1) cos (x y) x 2t
=

= cos ((1 + y) (+2)) y 2 cos (1 + t) t 2 (1 + t) 2t.

144

29.
1
1
+
xy
y
1
= 2
yx

w =

and

w
x

1
1
w
= 2 2.
y
xy
y

Also,
x = t2 + 1

dx
dt

= 2t

and
y = t4 + 3

dy
dt

= 4t 3 .

Hence,
dw
dt

w dx
w dy

x dt
y dt
1
1
1
= 2 2t + 2 2 4t 3
yx
xy
y
2t
4t 3
=

2
t4 + 3 t2 + 1
t2 + 1 t4 + 3
=

30. Let
4

F(x, y) = yx 3 + y 3 144.
Then
dy
dx

=
=

Fx
Fy

1
4
yx 3
3

1
x 4/3 + 43 y 3

so
dy
dx

(x,y)=(8,8)

4
8 (2)
= 3
16 + 43 2
8
=
.
7

4t 3
t4 + 3

145
31.
z
u

z x
z y

x u y u
= (y cos(x y) + sin y)2u + (x cos(x y) + x cos y)v.
=

If u = 0 and v = 1, then x = u 2 + v 2 = 1 and y = uv = 0. Then


z
u

32.

= (0 cos 0 + sin 0) 0 + (1 cos 0 + 1 cos 0) 1

= 2.

g x
g y
g
=

+
.
v
x v
y v

33.
f (x, y) = tan(x y) cot x
f

= sec2 (x y).x
y
2 y
2 = 2 sec(x y). sec(x y) tan(x y).x.x
y
= 2x 2 . sec2 (x y). tan(x y).
34.
y = sin(x y)

y sin(x y) = 0.
Let

F(x, y) = y sin(x y).


Then
dy
dx

Fx
Fy
cos (x y) y
=
1 cos (x y) x
y cos (x y)
=
.
1 x cos (x y)
=

35.
w
v

w x
w y

x v
y v
= 2(2x 2 + 2y 2 2)(4x)(2) + 2(2x 2 + 2y 2 2)(4y)(4).

If u = 0 and v = 1, then

x =0+2+2=4

MAT1512/1

146
and
y = 0 4 + 2 = 2
so that
w
v

= 2 (32 + 8 2) (16) (2) + 2 (32 + 8 2) (8) (4)


= 0.

36.
w = sin(x y)
w
= cos(x y).y

x
and
w
= cos(x y).x.
y
Also,
x = 1+t
dx

= 1
dt
and
y = t2
dy
= 2t.

dt
We have
dw
dt

w dx
w dy

x dt
y dt
= cos(x y) y 1 cos(x y) x 2t
=

= cos((1 + t)t 2 ) t 2 cos((1 + t)t 2 ) (1 + t)2t.


37. Put
F(x, y) = x y + y 2 3x 3.
Then

y3
Fx
dy
=
=
.
dx
Fy
x + 2y

Hence,
dy
dx

(x,y)=(1,1)

13
= 2.
1 + 2

147

MAT1512/1

38.
w
r

w x
w y

x r
y r
= 2 cos(2x y) 1 + ( cos(2x y)) s.

If r = and s = 0, then x = and y = 0, so that


w
= 2 cos(2) 1 cos(2).0 = 2.
r
39. Let
F(x, y) = 7y 4 + x 3 y + x 4.
Then
Fx
Fy
3x 2 y + 1
=
.
28y 3 + x 3

dy
dx

Hence

40.

dy
dx

(x,y)=(4,0)

1
.
64

(a) Denote derivatives with respect to time by a dot. If


V = abc,
then
V
V V
b+
a +
c
a
b
c
+ abc.
= abc
+ a bc

Substitute the given values:


V

= (2)(9)(5) + (13)(5)(5) + (13)(9)(2)


= 90 325 + 234

= 1.
Therefore the volume decreases at 1 cm3 /sec.
(b) Differentiate with respect to time. If
S = 2ac + 2bc + 2ab,
then
S S
S
a +
c
b+
a
b
c
= (2c + 2b)a + (2c + 2a)b + (2a + 2b)c.

S =

148
Substitute the given values:
S = (2 5 + 2 9)(2) + (2 5 + 2 13)(5) + (2 13 + 2 9)(2)
= 36.
The area decreases at a rate of 36 cm2 /sec.
(c) According to the Theorem of Pythagoras
1

D = (a 2 + b2 + c2 ) 2 .
Differentiate with respect to time:
D =
=
Thus D (13, 9, 5) =
=
=
=

1 2
1

(a + b2 + c2 ) 2 (2a a + 2bb + 2cc)


2
a a + bb + cc
.

a 2 + b2 + c 2
(13) (2) + (9) (5) + (5) (2)
(from the given values)
(13)2 + (9)2 + (5)2
26 45 + 10

169 + 81 + 25
9

275
9
.
5 11

Therefore the diagonal decreases at 9/ 5 11 cm/sec.

149

MAT1512/1

Key points
In this chapter we have introduced you to specific solutions of simple, first-order differential equations. You should
now be comfortable with solving some basic growth and decay problems. We have also introduced the notion of
partial derivatives, where we have seen how to approach scientific problems involving functions of several variables,
by changing one of the variables at a time, and keeping the remaining variable(s) fixed.
At this stage you should be able to:

solve first-order differential equations with initial values


solve basic real-life problems involving exponential growth and decay
determine the partial derivatives of functions of several variables
Continue practising solving problems until you have mastered the basic techniques! Go through the section For
your review at the end of each chapter to consolidate what you have learnt. Also use other textbooks and the
internet.

150

APPENDIX
The purpose of the Appendix is to help you to make the transition from high school to the more advanced first-year
calculus module MAT1512.

I.
II.
III.

Sequences and summation (sigma) notation


Mathematical induction
The Binomial Theorem

I. Sequences and summation notation


A sequence is a list of numbers written in a specific order. Sequences have many applications.

To read: Smith and Minton, section 4.2, pp 354359

Outcomes:
After studying this section you should:

(a) know the definition of a sequence.

(b) know what is meant by


a recursive sequence

a partial sum of a sequence


sigma notation

(c) be able to
find the terms of a sequence

151

MAT1512/1

find the n th term of a sequence

find the partial sum of a sequence

use sigma notation


Examples:

I. 1.1
Write the first four terms of the sequence given by the formula in each case.

(a) c j = 3

1
10

j1

Solution:

(b) an =

1
10

11

c1 = 3

1
10

21

c2 = 3

1
10

31

c3 = 3

1
10

41

c4 = 3

1
10

=3

1
10

=3

1
10

=3

1
10

=3

= 3,
=

3
,
10

3
100

3
.
1000

(1)n+1
n+3

Solution:

(c) xk =

a1 =

(1)1+1
1
1
(1)2+1
= , a2 =
=
1+3
4
2+3
5

a3 =

1
1
(1)3+1
(1)4+1
= , a4 =
= .
3+3
6
4+3
7

k
k+1

k+1
k

Solution:
x1 =
x2 =
x3 =
x4 =

1
1+1
2
2+1
3
3+1
4
4+1

1+1
1
2+1

2
3+1

3
4+1

1
2
2
=
3
3
=
4
4
=
5
=

3
2= ,
2
3
5
= ,
2
6
4
7
= ,
3
12
5
9
= .
4
20

152

I. 1.2 Find the ninth and tenth terms of: 0, 4, 0, . . . ,

2n + (2)n
,...
n

Solution:
an =
Then a9 =

2n + (2)n
n
29 + (2)9
29 29
0
=
= = 0 and
9
9
9

210 + (2)10
210 + 210
2 210
2048
1024
=
=
=
=
.
10
10
10
10
5
3
.
I. 1.3 Find the first six terms of the sequence defined by: a1 = 6 and an =
an1
a10 =

Solution:
a1 = 6, a2 =
a3 =
a4 =
a5 =
a6 =

3
a31
3
a41
3
a51
3
a61

3
a21

3
1
= ,
6
2

3
3
= 1 = 6,
a2
2

1
3
3
= = ,
a3
6
2

3
3
= 1 = 6,
a4
2

3
3
1
= = .
a5
6
2

I. 1.4 Find the sum of the first five terms of the sequence given by the formula in each case.
(a) ak = (1)k

1
k

Solution:
5
k=1

ak = a1 + a2 + a3 + a4 + a5
1
1
1
1
1
= (1)1 + (1)2 + (1)3 + (1)4 + (1)5
1
2
3
4
5
= 1 +
=

47
.
60

1 1 1 1
+
2 3 4 5

153
(b) bi = i 3
Solution:
5
i=1

i 3 = b1 + b2 + b3 + b4 + b5
= 13 + 23 + 33 + 43 + 53

= 1 + 8 + 27 + 64 + 125

= 225.
8

(c) Find
n=0

1
.
2n

xn where xn =

Solution:
8
n=0

xn = x0 + x1 + x2 + x3 + x4 + x5 + x6 + x7 + x8
=

1
1
1
1
1
1
1
1
1
+ 1+ 2+ 3+ 4+ 5+ 6+ 7+ 8
0
2
2
2
2
2
2
2
2
2

= 1+
=

1 1 1
1
1
1
1
1
+ + +
+
+
+
+
2 4 8 16 32 64 128 256

511
.
256

I. 1.5 Evaluate each of the following:


6

(a)

(5k)
k=1

Solution:
6
k=1

(5k) = 5.1 + 5.2 + 5.3 + 5.4 + 5.5 + 5.6


= 5 + 10 + 15 + 20 + 25 + 30

= 105.
7

(1)k

(b)
k=1

Solution:
7
k=1

(1)k = (1)1 + (1)2 + (1)3 + (1)4 + (1)5 + (1)6 + (1)7


= 1 + 1 1 + 1 1 + 1 1
= 1.

MAT1512/1

154
3

(c)
n=1

n+1
n

n
n+1

Solution:
3
n=1

n
n+1

n
n+1

1
2
3
1+1
2+1
3+1

1
1+1
2
2+1
3
3+1

= 2
=

1 3 2 4 3
+ +
2 2 3 3 4

35
.
12

I. 1.6 Rewrite each series using sigma notation.

(a) 4 + 8 + 12 + 16 + 20 + 24
Solution:
4 + 8 + 12 + 16 + 20 + 24 = 4 1 + 4 2 + 4 3 + 4 4 + 4 5 + 4 6
6

(4k).
k=1

(b) 4 2 + 0 + 2 + 4 + 6 + 8
Solution:
4 2 + 0 + 2 + 4 + 6 + 8 = 2(2) + 2(1) + 2 0 + 2 1 + 2 2 + 2 3 + 2 4
4

(2k).
k=2

155

MAT1512/1

II. Mathematical induction


We consider here a special kind of proof method called mathematical induction.
To read: Smith and Minton, section 4.2, pp 359360
Outcomes:
After studying this section you should

(a) know what the principle of mathematical induction is

(b) be able to prove using mathematical induction that a statement P (n) is true for all natural numbers n

Examples:
II. 2.1 Use mathematical induction to prove the following statement for all positive integers n :
1 + 4 + 7 + . . . + (3n 2) =

n(3n 1)
.
2

1 + 4 + 7 + . . . + (3n 2) =

n(3n 1)
2

Solution:
Let P (n) be the statement

for any positive integer n.


We first show that P (1) is true.
LHS of () = 3 1 2
= 1.

1(3 1 1)
2
2
=
2
= 1.

RHS of () =

Thus P (1) is true.


Suppose that P (k) is true for any k, that is, we assume
1 + 4 + 7 + . . . + (3k 2) =

k(3k 1)
.
2

()

156
We want to prove that P (k + 1) is true, in other words:
1 + 4 + 7 + . . . + (3k 2) + (3(k + 1) 2)
=

(k + 1) (3(k + 1) 1)
2

(k + 1)(3k + 2)
.
2

Now
1 + 4 + 7 + . . . + (3k 2) + (3(k + 1) 2)
=

k(3k 1)
+ (3k + 1)
2

1
[k(3k 1) + 2(3k + 1)]
2

1
3k 2 k + 6k + 2
2

1 2
(3k + 5k + 2)
2

(k + 1)(3k + 2)
.
2

Since both conditions of the principle of mathematical induction have been satisfied, it follows that
1 + 4 + 7 + . . . + (3n 2) =

n(3n 1)
2

is true for all integers n 1.


II. 2.2 Prove that
1 2 + 2 3 + 3 4 + . . . + n(n + 1) =

n(n + 1)(n + 2)
for all n N.
3

Solution:
We must verify that the equation is true for n = 1.
The LHS is 1(1 + 1) = 2.

1(1 + 1)(1 + 2)
= 2.
3
So the equation is true for n = 1.

The RHS is

Suppose that the equation is true for n = k, that is


1 2 + 2 3 + 3 4 + . . . + k(k + 1) =

k(k + 1)(k + 2)
3

()

157

MAT1512/1

We now use the assumption () to deduce that the equation is true for n = k + 1, in other words:
1 2 + 2 3 + 3 4 + . . . + k(k + 1) + (k + 1) [(k + 1) + 1]
(k + 1) [(k + 1) + 1] [(k + 1) + 2]
.
3

=
Now,

LHS = 1 2 + 2 3 + 3 4 + . . . + k(k + 1) + (k + 1) [(k + 1) + 1]


=

k(k + 1)(k + 2)
+ (k + 1) [(k + 1) + 1]
3

= (k + 1)

k(k + 2)
+ (k + 1) + 1
3

= (k + 1)

k 2 + 2k + 3k + 6
3

= (k + 1)

k 2 + 5k + 6
3

(k + 1)(k + 2)(k + 3)
3

(k + 1) [(k + 1) + 1] [(k + 1) + 2]
3

= RHS.
Since the equation is true for n = 1, and if it is true for n = k, then it is true for n = k + 1. We conclude by
mathematical induction that the equation is true for all integers n 1.
II. 2.3 Prove that
3 + 32 + 33 + . . . + 3n =

3n+1 3
for all integers n 1.
2

Solution:
We must verify that the equation is true for n = 1.
The LHS is 31 = 3.

31+1 3
=3
2
So the equation is true for n = 1.
The RHS is

Suppose that the equation is true for n = k, that is


3 + 32 + 33 + . . . 3k =

3k+1 3
2

()

158
We now use the assumption () to deduce that the equation is true for n = k + 1, in other words:
3 + 32 + 33 + . . . + 3k + 3k+1 =

3(k+1)+1 3
.
2

Now,

LHS = 3 + 32 + 33 + . . . + 3k + 3k+1
=

3k+1 3
+ 3k+1
2

3k+1 3 + 2 3k+1
2

3k+1 (1 + 2) 3
2

3k+1 3 3
2

3(k+1)+1 3
2

= RHS.
Since the equation is true for n = 1, and if it is true for n = k, then it is true for n = k + 1. We conclude by
mathematical induction that the equation is true for all integers n 1.
II. 2.4 Use mathematical induction to prove that
3
4

<

3
for n 2.
4

Solution:
Our starting point is n = 2, so we first verify that the inequality is true for n = 2.
LHS is

3
4

3 3
3
3
< = RHS since 0 < < 1.
4 4
4
4

So the inequality is true for n = 2.


Suppose that the inequality is true for n = k, that is
3
4

<

3
4

We now use the assumption () to deduce that the inequality is true for n = k + 1, in other words
3
4

k+1

3
< .
4

()

159

MAT1512/1

Now,
3
4

k+1

LHS =

3
4

3
4

<

3 3

4 4

by ()

<

3
4

by the case n = 2

= RHS.
Therefore if the inequality is true for n = k, it is true for n = k + 1. By mathematical induction we conclude that it
is true for all positive integers n 2.

III. The Binomial Theorem


To read: Smith and Minton, section 2.3, pp 171172 and section 8.8, p 691. It is not necessary to learn the proof of
the Binomial Theorem (3.4) on p 171, but you may know it as another example of proving by using mathematical
induction.

Outcomes: After studying this section you should

(a) know what is meant by


binomial expansion

binomial coefficients

Pascals triangle

160
(b) be able to
expand a polynomial using Pascals Triangle

expand a polynomial using the Binomial Theorem


find the general term of a binomial expansion

find a particular term in a binomial expansion

Examples:
III. 3.1 Expand and simplify:

(a) (3x 2)4


Solution:
We use the Binomial Theorem with a = 3x and b = 2,
(3x)4 + 4(3x)3 (2) +

43
432
(3x)2 (2)2 +
(3x) (2)3 + (2)4
12
123

= 81x 4 216x 3 + 216x 2 96x + 16.

(b)

2
x
+
2
y

Solution:
We use the Binomial Theorem with a =
2
x
+
2
y

x
2

x
2
and b = ,
2
y

5 x
+
1 2

5432 x
+
1234 2
=

2
y
2
y

54 x
+
12 2
4

2
y

2
y

543 x
+
123 2

x3
1 5 5 x4
x2
x
32
x +
+ 5 2 + 20 3 + 40 4 + 5 .
32
8 y
y
y
y
y

2
y

161

(c)

1
2a 2
a

MAT1512/1

Solution:
1
2a 2
a

1
6
= (2a) + (2a)5 2
1
a
6

1
6543
+
(2a)2 2
1234
a

65
1
+
(2a)4 2
12
a
4

= 64a 6 192a 3 + 240 160

1
654
+
(2a)3 2
123
a

65432
1
+
(2a) 2
12345
a

1
1
1
1
+ 60 6 12 9 + 12 .
3
a
a
a
a

III. 3.2 Find the x 5 y 3 term in the expansion of (2x y)8 .


Solution:
The general term in the binomial expansion is
8
8 8r 8r
(2x)8r (y)r =
2 x (1)r y r .
r
r
We need 8 r = 5, that is, r = 3.
The required term is
876 5 5 3
8 83 83
2 x (1)3 y 3 =
2 x y
123
3
= 1792 x 5 y 3 .
8

III. 3.3 Find the a b term in the expansion

a b2
+
2
3

10

Solution:
The general term in the binomial expansion is
10
r

a
2

10r

b2
3

10 r10 10r r 2r
2
a
3 b .
r

We need 10 r = 8, that is, r = 2.


The required term is
5 8 4
10 210 102 2 2 2 10 9 8 8 2 4
a
3 b =
2 a 3 b =
a b .
2
12
256
2

1
+ 2
a

162
III. 3.4 Find the term that contains y 10 in the expansion of (x 2y 2 )8 .
Solution:
The general term in the binomial expansion is
8 8r
8 8r
x
(2y 2 )r =
x
(2)r y 2r .
r
r
We need 2r = 10, that is, r = 5.
The required term is
8 85
x
(2)5 y 25 =
5

876 3
x (2)5 y 10
123

= 1792 x 3 y 10 .

Extra reading:
Stewart, J, Redlin, L, and Watson, S. Precalculus, 5th edition (or 4th or 3rd edition).
Chapter 11: Sections 11.1, 11.2, 11.3, 11.5 and 11.6 (or chapter 10.6 in the 4th and
3rd editions)

Anda mungkin juga menyukai